Sie sind auf Seite 1von 113

WWW.INSIGHTSONINDIA.

COM WWW.INSIGHTSIAS.COM
TEST - 3

2016 Test Series

Test - 3

1. The difference between a Sanctuary and a National Park mainly lies in

a) Area, size and government control

b) PRIs v/s state government control

c) Rights of people living inside

d) Regulation of poaching and hunting

Solution: c)

Explanation: Consider this. Anamudi Shola National Park lies only in


7.5 Km2 whereas Pulicat Lake Bird Sanctuary lies in more than 327 km2.
On the other hand, Temple Island sanctuary in Andaman is in merely 1
km2 area, whereas Mahatma Gandhi Marine National Park in Andaman
lies in more than 281 Km2. So, area and size is not the essential
difference between both. Option (a) is wrong.

PRis do not control the activities in sanctuaries or national parks. Either


the state governments, or the National Board of Wildlife does so. Option
(c) is wrong.

Poaching and hunting are regulated in both. So, option (d) is also wrong.

Learning: The difference between a Sanctuary and a National Park


mainly lies in the vesting of rights of people living inside. Unlike a
Sanctuary, where certain rights can be allowed, in a National Park, no
rights are allowed.

No grazing of any livestock shall also be permitted inside a National Park


while in a Sanctuary, the Chief Wildlife Warden may regulate, control or
prohibit it.

In addition, while any removal or exploitation of wildlife or forest


produce from a Sanctuary requires the recommendation of the State
Board for Wildlife, removal etc., from a National Park requires
recommendation of the National Board for Wildlife (However, as per


PRELIMS TEST SERIES 2016 MOCK TEST - 1
INSIGHTS

WWW.INSIGHTSONINDIA.COM WWW.INSIGHTSIAS.COM
TEST - 3

orders of Hon’ble Supreme Court dated 9th May 2002 in Writ Petition
(Civil) No. 337 of 1995, such removal/ exploitation from a Sanctuary also
requires recommendation of the Standing Committee of National Board
for Wildlife).

Q Source: Improvisation: Chapter 7: Science 8th NCERT: Page 79

http://www.moef.nic.in/downloads/public-information/protected-area-
network.pdf

2. Wular Lake, the largest freshwater lake in India, has been formed due
to

a) Flooding of many small riverine islands

b) Ox-bow lakes

c) Filling of glacier basin

d) Tectonic activity

Solution: d)

Learning: Some lakes are the result of the action of glaciers and ice
sheets, while others have been formed by wind, river action, and human
activities.

A meandering river across a flood plain forms cut-offs that later develop
into ox-bow lakes. Spits and bars form lagoons in the coastal areas, eg
the Chilika lake, the Pulicat lake, the Kolleru lake.

Most of the fresh water lakes are in the Himalayan region. They are of
glacial origin.

In other words, they formed when glaciers dug out a basin, which was
later filled with snowmelt.

The Wular lake in Jammu and Kashmir, in contrast, is the result of the
tectonic activity. It is the largest freshwater lake in India.

Q Source: Chapter 3: Contemporary India – I: Page 22


PRELIMS TEST SERIES 2016 MOCK TEST - 1
INSIGHTS

WWW.INSIGHTSONINDIA.COM WWW.INSIGHTSIAS.COM
TEST - 3

3. Which of the following can be called as ‘factors of production’?

1. Human labour

2. Land

3. Consumer goods

4. Capital goods

Select the correct answer using the codes below.

a) 1, 2 and 3 only

b) 2, 3 and 4 only

c) 1 only

d) 1, 2 and 4 only

Solution: d)

Explanation: Human labour and land contribute to production. So,


statements 1 and 2 are correct.

Capital goods are machines which are used to produce other goods. So,
statement 4 is also correct.’

Consumer goods are finished products. They are not factors of


production, but products. So, statement 3 is wrong.

Learning: It describes the inputs that are used in the production of


goods or services in the attempt to make an economic profit. The factors
of production include land, labor, capital and entrepreneurship.

Capital can be further divided into fixed (machines etc) and working
capital (money, raw material etc).

Q Source: Chapter 1: Economics 9th NCERT: Page 2

4. Consider the following statements.


PRELIMS TEST SERIES 2016 MOCK TEST - 1
INSIGHTS

WWW.INSIGHTSONINDIA.COM WWW.INSIGHTSIAS.COM
TEST - 3

Assertion (A): Coastal regions in India do not have irrigation


facilities.

Reason (R): Farming in coastal areas is completely rain-fed due to


high rainfall near the sea.

In the context of the above, which of these is correct?

a) A is correct, and R is an appropriate explanation of A.

b) A is correct, but R is not an appropriate explanation of A.

c) A is correct, but R is incorrect.

d) Both A and R are incorrect.

Solution: d)

Explanation: Rainfall in coastal areas is not adequate, especially some


of the eastern coasts. This is why coastal areas have a well-developed
irrigation infrastructure. Statement A is wrong, and so is R.

Learning: Groundwater system irrigation is the largest in India. In


2010, only about 35% of total agricultural land in India was reliably
irrigated. About 2/3rd cultivated land in India is dependent on
monsoons.

Riverine plains in India are well-irrigated. In contrast, plateau regions


such as the Deccan plateau have low levels of irrigation.

Q Source: Chapter 1: Economics 9th NCERT: Page 3

5. Consider the following statements about Food and Agriculture


Organization (FAO).

1. It is an agency of the United Nations

2. It serves only developing and Least Developed Countries


(LDCs).


PRELIMS TEST SERIES 2016 MOCK TEST - 1
INSIGHTS

WWW.INSIGHTSONINDIA.COM WWW.INSIGHTSIAS.COM
TEST - 3

3. It created the Codex Alimentarius Commission to develop food


standards.

Select the correct answer using the codes below.

a) 2 and 3 only

b) 1 only

c) 1 and 3 only

d) 2 only

Solution: c)

Explanation & Learning: Its world headquarters are located in Rome.


It leads international efforts to defeat hunger.

Serving both developed and developing countries, FAO acts as a neutral


forum where all nations meet as equals to negotiate agreements and
debate policy.

FAO is also a source of knowledge and information, and helps


developing countries and countries in transition modernize and improve
agriculture, forestry and fisheries practices, ensuring good nutrition and
food security for all.

FAO and the World Health Organization created the Codex Alimentarius
Commission in 1961 to develop food standards, guidelines and texts such
as codes of practice under the Joint FAO/ WHO Food Standards
Programme.

Q Source: Improvisation: Chapter 4: Economics 9th NCERT: Page 42


(Right side box)

6. Use of chemical fertilizers is discouraged because

1. Its minerals take long time to become available to plants.


PRELIMS TEST SERIES 2016 MOCK TEST - 1
INSIGHTS

WWW.INSIGHTSONINDIA.COM WWW.INSIGHTSIAS.COM
TEST - 3

2. It may kill soil micro-organisms.

3. Agricultural run-off may cause eutrophication of fresh water


bodies

4. High levels of fertilizer may cause the breakdown of the


symbiotic relationships between plant roots and mycorrhizal fungi.

Select the correct answer using the codes below.

a) 2 only

b) 2 and 3 only

c) 1, 3 and 4 only

d) 2, 3 and 4 only

Solution: d)

Explanation & Learning: Chemical fertilizers provide minerals which


dissolve in water and are immediately available to plants. So, statement 1
is wrong.

But these minerals may not be retained in the soil for long. They may
escape from the soil and pollute groundwater, rivers and lakes. So,
statement 3 is correct.

Chemical fertilizers can also kill bacteria and other microorganisms in


the soil. This means some time after their use, the soil will be less fertile
than ever before, and some symbiotic relations based on micro-
organisms will be broken. So, statement 2 and 4 are correct.

It has been explained very well here.


https://en.wikipedia.org/wiki/Fertilizer#Environmental_effects

UPSC 2015 Prelims question statements on Nitrogen fertilizers was


straight lifted from here.

Q Source: Improvisation: Chapter 1: Economics 9th NCERT: Page 6

6. Apart from Article 21A, which of the following constitutional articles


have a bearing on education?


PRELIMS TEST SERIES 2016 MOCK TEST - 1
INSIGHTS

WWW.INSIGHTSONINDIA.COM WWW.INSIGHTSIAS.COM
TEST - 3

1. Article 3

2. Article 13

3. Article 28

4. Article 30

5. Article 51A

6. Article 243G

Select the correct answer using the codes below.

a) 1, 2 and 6 only

b) 3, 4 and 5 only

c) 2, 3, 4, 5 and 6 only

d) All of the above

Solution: c)

Explanation: You should remember important constitutional articles.

Article 3 is about creation/alteration etc of states. Hence, statement 1 is


wrong.

Article 13 is about laws inconsistent with or in derogation of the


fundamental rights (read education here). So, statement 2 is correct.

Article 28 is about the freedom as to attendance at religious instruction


or religious worship in certain educational institutions. So, statement 3
is correct.

Article 30 is about the right of minorities to establish and administer


educational institutions. So, statement 4 is also correct.

Article 51A is set of fundamental duties: one of which is to develop the


scientific temper, humanism and the spirit of inquiry and reform. So,
statement 5 is also correct.

Article 243G talks about the powers/authority/responsibility of


Panchayats, which also covers school education. So, statement 6 is also
correct.


PRELIMS TEST SERIES 2016 MOCK TEST - 1
INSIGHTS

WWW.INSIGHTSONINDIA.COM WWW.INSIGHTSIAS.COM
TEST - 3

Learning & Q Source: UPSC had framed its 2011 question on


education from here itself. We have improvised on education policy
mentioned in NCERT Page 21: Economics: 9th

http://mhrd.gov.in/constitutional-provision

Check other tabs to the right under constitutional provisions.

7. Who is the ‘Visitor’ of Central Universities in India?

a) Prime Minister of India

b) President of India

c) An Academician appointed by Union Ministry of Human Resource


Development (MHRD)

d) Chairman, University Grants Commission (UGC)

Solution: b)

Explanation & Learning: The President of India is the Visitor of 114


central institutions. This includes Central Universities, NITs, IITs,
IISERs etc. under the administrative control of Ministry of HRD. Some
of the institutions are under the administrative control of Ministry of
Shipping, Ministry of External Affairs, Ministry of Agriculture, Ministry
of Sports, Ministry of Textiles and Ministry of Chemicals & Fertilizers.
The powers of the President as a Visitor of these institutions are defined
in their respective Acts.

Some of his powers are to inspect the university, chair conference of


Directors, power to annul unconstitutional provisions/rules w.r.t to the
Universities.

Q Source: Improvisation: NCERT 9th: Economics: Education topics:


http://mhrd.gov.in/higher_education

8. Which of the following are MDG 2015 targets which have been
achieved by India?


PRELIMS TEST SERIES 2016 MOCK TEST - 1
INSIGHTS

WWW.INSIGHTSONINDIA.COM WWW.INSIGHTSIAS.COM
TEST - 3

1. Universal primary education

2. Reverse the spread of HIV/AIDS

3. Eliminate poverty from urban areas

Select the correct answer using the codes below.

a) 1 only

b) 1 and 3 only

c) 2 only

d) 2 and 3 only

Solution: c)

Explanation: Eliminating poverty from urban areas is not a target


under MDG 2015. Hence, statement 3 is incorrect.

India has neither been able to nor likely to achieve universal primary
education by the end of 2015, as per a Ministry report (given below). So,
statement 1 is also wrong.

MDG 6: Combat HIV/AIDS, malaria and other diseases – under this


Target 7 – diseases have halted by 2015 and begun to reverse the spread
of HIV/AID. So, statement 2 is correct.

Learning & Q Source: The appendix 1 of the following report (Page


158- 163) is a summary of India’s MDG achievements submitted to the
United Nations.
http://mospi.nic.in/Mospi_New/upload/mdg_26feb15.pdf

Improvisation: MMR and IMR used in NCERT blue boxes: 9th


Economics: Page 23

9. How has India benefitted from signing the Indus water treaty with
Pakistan?

a) The treaty declares India to be the owner of Indus water resources.


PRELIMS TEST SERIES 2016 MOCK TEST - 1
INSIGHTS

WWW.INSIGHTSONINDIA.COM WWW.INSIGHTSIAS.COM
TEST - 3

b) The treaty bans Pakistan from constructing and embankments or


dams on the Indus river system.

c) As per the treaty, to obtain water from Indus, Pakistan is obliged to


share it irrigation infrastructure with India.

d) None of the above

Solution: d)

Explanation: According to this agreement; three rivers: ravi, satluj and


beyas were given to india and the other three: sindh, jehlam and chenab
were given to Pakistan. So, option (a) is clearly wrong.

The treaty does not ban Pakistan from creating dams, as it is a lower
riparian state. India’s embankment construction activities may be
questioned by Pakistan, if it threatens its water resources. Recent
Kishenganga water dispute is a case in point. Option (b) is incorrect.

Option (c) was not provided for by the treaty.

Learning: A transition period of 10 years was permitted in which India


was bound to supply water to Pakistan from these rivers until Pakistan
was able to build the canal system for utilization of waters of Jhelum,
Chenab and the Indus itself, allocated to it under the treaty.

Pakistan also received one-time financial compensation for the loss of


water from the eastern rivers. India has now secured full rights for use of
the waters of the three rivers allocated to it. The treaty resulted in
partitioning of the rivers rather than sharing of their waters.

The countries agree to exchange data and co-operate in matters related


to the treaty. For this purpose, treaty creates the Permanent Indus
Commission, with a commissioner appointed by each country.

Q Source: Chapter 3: Contemporary India – I: Page 20

10. Which of the following organizations publish the ‘International


Poverty Line’?

a) United Nations Development Program (UNDP)


PRELIMS TEST SERIES 2016 MOCK TEST - 1
INSIGHTS

WWW.INSIGHTSONINDIA.COM WWW.INSIGHTSIAS.COM
TEST - 3

b) World Food Programme (WFP)

c) International Labour Organization (ILO)

d) World Bank

Solution: d)

Explanation & Learning: The World Bank has been publishing


poverty line since 1990 World Development Report (WDR) on Poverty.
It has anchored its international poverty lines to the national poverty
lines used in the poorest countries.

In 2008, the World Bank came out with a figure (revised largely due to
inflation) of $1.25 at 2005 purchasing-power parity (PPP). The common
international poverty line has in the past been roughly $1 a day

In 2015 World Bank updated international poverty line to US $1.90 a


day.

Determining the poverty line is usually done by finding the total cost of
all the essential resources that an average human adult consumes in one
year.[

Note: World Bank should not be confused with the United Nations
World Bank Group, a member of the United Nations Economic and
Social Council.

Q Source: Improvisation: Chapter 3: Economics 9th NCERT: Page 36

11. The Mahatma Gandhi National Rural Employment Guarantee Act


(MGNREGA) intends to enhance livelihood security in rural areas by
providing at least 100 days of guaranteed wage employment in a
financial year to willing households. Which of the following statements
about it is INCORRECT?

a) Any type of skilled or unskilled work can be allotted under


MGNREGA.

b) One of the aims of MGNREGA is to create durable assets.


PRELIMS TEST SERIES 2016 MOCK TEST - 1
INSIGHTS

WWW.INSIGHTSONINDIA.COM WWW.INSIGHTSIAS.COM
TEST - 3

c) MGNREGA is to be implemented mainly by gram panchayats (GPs) as


the involvement of contractors is banned.

d) If work is not provided within 15 days of applying, applicants are


entitled to an unemployment allowance.

Solution: a)

Explanation: Only unskilled manual work is allotted under


MGNREGA. So, statement 1 is incorrect.

MGNREGA aims at creating durable assets like roads, canals etc. So,
statement 2 is correct.

Contractors cannot be used in allotting MGNREGA work. So, statement


3 is correct.

It aims to guarantee the 'right to work'. Employment is to be provided


within 5 km of an applicant's residence, and minimum wages are to be
paid. If work is not provided within 15 days of applying, applicants are
entitled to an unemployment allowance. Thus, employment under
MGNREGA is a legal entitlement. So, statement 4 is also correct.

Learning: The statute is hailed by the government as "the largest and


most ambitious social security and public works programme in the
world". In its World Development Report 2014, the World Bank termed
it a "stellar example of rural development. Work is provided to every
household whose adult members volunteer to do unskilled manual work.

Apart from providing economic security and creating rural assets,


NREGA can help in protecting the environment, empowering rural
women, reducing rural-urban migration and fostering social equity,
among others.

Q Source: Improvisation: Chapter 3: Economics 9th NCERT: Page 39

12. Apart from Supreme Court, writs to enforce fundamental rights can
be issued by

1. District Courts


PRELIMS TEST SERIES 2016 MOCK TEST - 1
INSIGHTS

WWW.INSIGHTSONINDIA.COM WWW.INSIGHTSIAS.COM
TEST - 3

2. National Police Commission

3. Lok Adalats

Select the correct answer using the codes below.

a) 1, 2 and 3 only

b) 1 and 3 only

c) 2 only

d) None of the above

Solution: d)

Explanation: The Parliament is authorized to extend the power to issue


writs to any other court other than the SC and the HCs. But it has not
extended the same to any other body in India. So, all statements are
wrong.

Lok adalat and District courts thus only solve disputes and do not
interfere pro-actively in matter of violations of fundamental rights.

National Police Commissions are constituted by government to reform


the police system in the country. So, statement 2 is also wrong.

Q source: Chapter 6: Democratic Politics: Page 108

13. Village Grain Banks Scheme is concerned with

a) Providing loans to small and marginal farmers in case of crop loss

b) Providing facility to borrow food grains in food scarce and calamity hit
areas

c) Giving loans for food supply chain creation in rural areas

d) Providing a single window mechanism for farmers to procure


agricultural inputs

Solution: b)


PRELIMS TEST SERIES 2016 MOCK TEST - 1
INSIGHTS

WWW.INSIGHTSONINDIA.COM WWW.INSIGHTSIAS.COM
TEST - 3

Explanation & Learning: Since 2004, it is being run by Department


of Food & Public Distribution. The main objective of the scheme was to
provide safeguard against starvation during the period of natural
calamity or during lean season when the marginalized food insecure
households did not have sufficient resources to purchase rations.

Such people in need of food grains will be able to borrow food grains
from the Village Grain Bank. The grain banks are to be set up in food
scarce areas like the drought prone areas, the hot and cold desert areas,
tribal areas and the inaccessible hilly areas which remain cut off because
of natural calamities like floods, etc.

These villages are to be notified by the concerned State


Government/Union Territory. The scheme envisages inclusion of all
willing BPL/AAY families in the villages to be identified by the State
Government in food deficit areas.

The quantity to be lent and the period of repayment is to be decided by


the Group themselves. Village Panchayat/Gram Sabha, Self Help Group
for NGOs etc are identified by the state governments and eligible for
running the Grain banks.

The scheme has been discontinued w.e.f. 2014.

Q Source: http://dfpd.nic.in/plan-schemes.htm

Improvisation Chapter 4: Economics 9th NCERT: Page 52

14. Wangala Festival is

a) a major religious procession of Khonds in Sikkim

b) annual post-harvest festival of the Garos tribes of North east India

c) a literary event commemorating the writings of vernacular language


authors of eastern India

d) None of the above

Solution: b)

Learning:


PRELIMS TEST SERIES 2016 MOCK TEST - 1
INSIGHTS

WWW.INSIGHTSONINDIA.COM WWW.INSIGHTSIAS.COM
TEST - 3

• The Wangala is a Garo post-harvest festival that marks the end of


the agricultural year. It is an act of thanksgiving to the sun god of
fertility, known as Misi-A-Gilpa-Saljong-Galapa.

• A nagara (a special drum used for calling the people on solemn


occasions) is beaten. The social aspect of the Wangala Festival goes
on in the villages for a number of days, with eating, drinking and
merrymaking.

• This is the most popular festival of the Garo Hills, and is held in
November, the precise date being fixed by the headman.

• The men and women dance in mirthful gaiety with the beating of
drums, blowing of the buffalo horn trumpets and bamboo flutes.
The men wear dhotis, half-jackets and turbans with feathers. The
women wear colourful dresses made of silk, blouses and a head-
wrap with feathers.

• The highlight of the festival is when 300 dancers and 100 drums
descend on the field in all their splendour in celebration.

Q Source: http://www.thehindu.com/todays-paper/tp-features/tp-
fridayreview/momentous-movements/article7135285.ece

http://www.thehindu.com/features/friday-review/music/rahman-team-
to-record-wangala-beat-for-cwg-ceremony/article47219.ece

15. In India, Minimum Support Prices (MSP) are announced for

1. Pulses

2. Oilseeds

3. Tea

4. Coffee

Select the correct answer using the codes below.

a) 1 and 2 only

b) 2 and 3 only


PRELIMS TEST SERIES 2016 MOCK TEST - 1
INSIGHTS

WWW.INSIGHTSONINDIA.COM WWW.INSIGHTSIAS.COM
TEST - 3

c) 1, 3 and 4 only

d) All of the above

Solution: a)

Explanation & Learning: You can see the list of crops here
http://eands.dacnet.nic.in/PDF/MSP17.06.2015.pdf

Minimum Support Price (MSP) is a form of market intervention by the


Government of India to insure agricultural producers against any sharp
fall in farm prices.

The minimum support prices are announced by the Government of India


at the beginning of the sowing season for certain crops on the basis of the
recommendations of the Commission for Agricultural Costs and Prices
(CACP).

The major objectives are to support the farmers from distress sales and
to procure food grains for public distribution. In case the market price
for the commodity falls below the announced minimum price due to
bumper production and glut in the market, govt. agencies purchase the
entire quantity offered by the farmers at the announced minimum price.

Q Source: Chapter 4: Economics 9th NCERT: Page 51

16. Gold Monetisation Scheme (GMS) 2015 is available to

a) Only Non-Resident Indians (NRIs)

b) Both NRIs and resident Indians

c) Only Indian citizens with income above a certain threshold

d) All resident Indians

Solution: d)

Learning: It was launched to reduce the physical demand for gold and
fish out 20,000 tonnes of the precious metal lying idle with households
and institutions.


PRELIMS TEST SERIES 2016 MOCK TEST - 1
INSIGHTS

WWW.INSIGHTSONINDIA.COM WWW.INSIGHTSIAS.COM
TEST - 3

• GMS 2015 will offer option to resident Indians to deposit their


precious metal and earn an interest on it and will replace the
existing Gold Deposit Scheme (GDS) 1999.

• All residents can invest in this scheme but are subjected to Know
Your Customer (KYC) Scheme.

• The tagline of the Scheme is, “Earn, while you secure”.

• As per Reserve Bank of India (RBI) directives all Scheduled


Commercial Banks (excluding Regional Rural Banks) will
implement GMS 2015.

Q Source: http://economictimes.indiatimes.com/topic/Gold-
Monetization-Scheme

17. Consider the following about Rhizobium.

1. It colonises legume plant’s root nodules.

2. It provides food to those symbiotic species which cannot


manufacture food.

3. It grows by the technique of vegetative propagation.

Which of the above is/are correct?

a) 1 only

b) 2 and 3 only

c) 1 and 3 only

d) 2 only

Solution: a)

Explanation: Rhizobium is a soil bacteria that fixes nitrogen, so it can’t


manufacture food. Hence, statement 2 is incorrect.

Only plants are capable of growth through vegetative propagation (i.e.


growing another full plant with parts of a plant). Hence, statement 3 is
incorrect.


PRELIMS TEST SERIES 2016 MOCK TEST - 1
INSIGHTS

WWW.INSIGHTSONINDIA.COM WWW.INSIGHTSIAS.COM
TEST - 3

Learning: Both bacteria and blue-green algae can fix nitrogen directly
from the atmosphere, but this is less vital to plant development than the
symbiotic relationship between the bacteria genus Rhizobium and
leguminous plants and certain trees and shrubs.

Some nitrogen-fixing bacteria, such as Azotobacter, Clostridium


pasteurianum, and Klebsiella pneumoniae, are free-living, whereas
species of Rhizobium live in an intimate association with leguminous
plants. Rhizobium organisms in the soil recognize and invade the root
hairs of their specific plant host, enter the plant tissues

In return for secretions (food) from their host that encourage their
growth and multiplication,

Q Source: Improvisation: Chapter 1: Science 8th NCERT: Page 6

18. Which authority in India maintains the Voter’s list for a particular
state?

a) Chief Electoral Officer of the State

b) District Magistrates of the concerned areas

c) Chief Secretary

d) Zila Parishad

Solution: a)

Explanation: In a democratic election, the list of those who are eligible


to vote is prepared much before the election and given to everyone. This
list is officially called the Electoral Roll and is commonly known as the
Voters’ List.

Election Commission of India has been transparent enough and has been
urging its CEOs in the states to set up their own web-sites too.

The CEO of a state is usually a senior IAS officer.

Q source: Improvisation: Chapter 4: Democratic Politics: Page 62


PRELIMS TEST SERIES 2016 MOCK TEST - 1
INSIGHTS

WWW.INSIGHTSONINDIA.COM WWW.INSIGHTSIAS.COM
TEST - 3

19. Which of these may NOT be a good reason to say that elections in a
certain country are democratic?

a) If it has very large voter base.

b) If an independent and powerful body supervises elections.

c) If all candidates have a fair chance of competing and winning in


elections.

d) If the losing parties peacefully accept the electoral verdict.

Solution: a)

Explanation: There may be confusion between option (a) and (c).


Option (a) is more appropriate because merely a large voter base does
not ensure successful and democratic elections. For e.g. Iraq may have a
large voter base, many agencies alleged that its 2003 elections were not
democratic. Malpractices, rigging etc can take place in countries with
large voter base. In such cases, elections will not be fair and democratic.

Options (c) and (d) show the political situation in the country. For e.g. if
in Pakistan, the incumbent government does not have any chances of
losing (as it controls state institutions), the elections may not be
considered democratic!

To ensure the above option (b) is necessary.

Q source: Chapter 4: Democratic Politics

20. Magnesium deficiency in plants can cause degradation of chlorophyll


in old leaves. In what type of soil is Magnesium deficiency most likely to
occur due to leaching?

a) Red soil with high loam content

b) Strongly acidic, light and sandy soils

c) Clayey soil which has high mineral adhesion

d) Any soil in areas of moderate rainfall

Solution: b)


PRELIMS TEST SERIES 2016 MOCK TEST - 1
INSIGHTS

WWW.INSIGHTSONINDIA.COM WWW.INSIGHTSIAS.COM
TEST - 3

Explanation: In strongly acidic, light, sandy soils magnesium can be


easily leached away. So, option (b) is correct.

In all other soils, where adhesion, denseness or less leaching occurs,


magnesium may not be leached. So, options (a), (c) and (d) are incorrect.

Learning: Magnesium is an essential macro nutrient found from 0.2-


0.4% dry matter and is necessary for normal plant growth.

Magnesium has an important role in photosynthesis because it forms the


central atom of chlorophyll. Therefore, without sufficient amounts of
magnesium, plants begin to degrade the chlorophyll in the old leaves

Low amounts of Mg lead to a decrease in photosynthetic and enzymatic


activity within the plants.

Plants deficient in magnesium also produce smaller, woodier fruits.

Excess potassium, generally due to fertilizers, further aggravates the


stress from the magnesium deficiency, as does aluminium toxicity.

Applying home-made compost mulch can prevent leaching during


excessive rainfall and provide plants with sufficient amounts of
nutrients, including magnesium

Q Source: Improvisation: Chapter 4: Science 8th NCERT: Page 52

21. International Covenant on Economic, Social and Cultural Rights does


NOT recognize which human right?

a) Right to work opportunity

b) Right to social security and insurance

c) Right to adequate housing

d) Right against indulgement in war by the State

Solution: d)

Explanation & Learning: ICESCR is a multilateral treaty adopted by


the UN General Assembly in 1966, coming into force since 1976.


PRELIMS TEST SERIES 2016 MOCK TEST - 1
INSIGHTS

WWW.INSIGHTSONINDIA.COM WWW.INSIGHTSIAS.COM
TEST - 3

It commits its parties to work toward the granting of economic, social,


and cultural rights (ESCR) to the Non-Self-Governing and Trust
Territories and individuals, including labour rights and the right to
health, the right to education, and the right to an adequate standard of
living. As of 2015, the Covenant has 164 parties. Six countries, including
the United States, have signed but not ratified the Covenant

The Covenant is monitored by the UN Committee on Economic, Social


and Cultural Rights.

Q source: Chapter 6: Democratic Politics: Page 110

22. Why is the colour of a candle flame yellow, whereas flame of a


kitchen stove is blue?

a) Temperature in candle flame is higher than that of kitchen stove

b) The amount of fuel burning per unit of time is very less in a candle as
compared to a kitchen stove

c) Fuel burns completely in a stove and has better exposure to oxygen


than in a candle

d) All of the above

Solution: c)

Explanation: The LPG gas in the stove is a Saturated Hydrocarbon.


Being a gas with large surface area, it gets sufficient oxygen for burning.

Paraffin wax in candles is unsaturated hydrocarbon. Since its a solid, it


does not burn completely due to insufficient oxygen reaching to the fuel
(inner portion). This causes a bright yellow flame to form and release
Carbon Dioxide on combustion.

Learning: In the most common type of flame, hydrocarbon flames, the


most important factor determining color is oxygen supply and the extent
of fuel-oxygen pre-mixing, which determines the rate of combustion and


PRELIMS TEST SERIES 2016 MOCK TEST - 1
INSIGHTS

WWW.INSIGHTSONINDIA.COM WWW.INSIGHTSIAS.COM
TEST - 3

thus the temperature and reaction paths, thereby producing different


color hues.

Different flame types of a Bunsen burner depend on oxygen supply. In


the diagram above, on the left a rich fuel with no premixed oxygen
produces a yellow sooty diffusion flame; but on the right a lean fully
oxygen premixed flame produces no soot and the flame color blue.

The same thing happens in a candle and a stove.

Q Source: Improvisation: Chapter 6: Science 8th NCERT: Page 70

23. A ‘by-election’ is held when

a) A party candidate during elections withdraws or dies

b) Vacancy caused by death or resignation of a legislator is to be filled


between general elections

c) The Election Commission of India thinks it to be in the best-interest of


the nation.

d) There is no decisive winner in a general election

Solution: b)

Explanation & Learning: In most cases these elections occur when


the incumbent died or resigned, but they also occur when the incumbent
becomes ineligible to continue in office (because of a recall,


PRELIMS TEST SERIES 2016 MOCK TEST - 1
INSIGHTS

WWW.INSIGHTSONINDIA.COM WWW.INSIGHTSIAS.COM
TEST - 3

ennoblement, criminal conviction, or failure to maintain a minimum


attendance).

Less commonly, these elections have been called when a constituency


election is invalidated by voting irregularities.

Q source: Chapter 4: Democratic Politics: Page 60

24. In a wildlife sanctuary, generally which of the following activities are


regulated?

1. Plantation and cultivation

2. Grazing

3. Tourism

4. Scientific research

Select the correct answer using the codes below.

a) 1 and 4 only

b) 2 and 3 only

c) 1 and 2 only

d) All of the above

Solution: d)

Explanation & Learning: To protect our flora and fauna and their
habitats, protected areas called sanctuaries, national parks and
biosphere reserves have been earmarked. Plantation, cultivation,
grazing, felling trees, hunting and poaching are prohibited there.

As per the Wildlife Act, The Chief Wildlife Warden may, on application,
grant to any person a permit to enter or reside in a sanctuary for all or
any of the following purposes, namely:

• Investigation or study of wildlife and purposes ancillary or


incidental thereto;
• Photography;


PRELIMS TEST SERIES 2016 MOCK TEST - 1
INSIGHTS

WWW.INSIGHTSONINDIA.COM WWW.INSIGHTSIAS.COM
TEST - 3

• Scientific research;
• Tourism;
• Transaction of lawful business with any person residing in the
sanctuary.

Q Source: Chapter 7: Science 8th NCERT: Page 78

25. Which of the following describes the ‘Mission’ of the Atomic Energy
Regulatory Board most appropriately?

a) Safeguarding public health and environment from hazards of nuclear


energy

b) Addressing energy security of the nations by establishing high end


nuclear power stations

c) Supervising quality research in matters of fundamental nuclear energy


to serve national interest

d) Oversee the technical operations of nuclear power centres in India

Solution: a)

Explanation & Learning: The Mission of the AERB is to ensure the


use of ionising radiation and nuclear energy in India does not cause
undue risk to the health of people and the environment.

It was established in 1983 by the President of India.

AERB is supported in its functions by a number of committees. Members


of all the AERB committees are recognized experts with long experience
in the relevant fields. The regulatory authority of AERB is derived from
the rules and notifications promulgated under the Atomic Energy Act,
1962 and the Environmental (Protection) Act, 1986.

Q Source: http://dae.nic.in/?q=node/36

26. Consider the following statements.


PRELIMS TEST SERIES 2016 MOCK TEST - 1
INSIGHTS

WWW.INSIGHTSONINDIA.COM WWW.INSIGHTSIAS.COM
TEST - 3

1. It lies in the Satpura Range of Madhya Pradesh.

2. It includes Bori sanctuary and Satpura national park.

3. It is a part of the World Network of Biosphere Reserves,


UNESCO.

The above statements refer to?

a) Panna Biosphere Reserve

b) Panchmari Biosphere Reserve

c) Achanakmar-Amarkantak Biosphere Reserve

d) Simlipal Biosphere Reserve

Solution: b)

Explanation & Learning: The conservation area was created in 1999


by the Indian government. It also houses Pachmarhi Sanctuary.

The reserve is composed primarily of forest habitats, and is an important


transition zone between the forest species of western and eastern India.

The endemic fauna includes chinkara, nilgai, wild dogs, the Indian Wolf,
bison, Indian giant squirrels, and flying squirrels.

Q Source: Chapter 7: Science 8th NCERT: Page 79


PRELIMS TEST SERIES 2016 MOCK TEST - 1
INSIGHTS

WWW.INSIGHTSONINDIA.COM WWW.INSIGHTSIAS.COM
TEST - 3

27. Why the difference between the durations of day and night is felt
hardly at Kanyakumari but not so much in Kashmir?

a) Due to difference of altitude of two places above Mean Sea Level


(MSL)

b) Due to difference in latitudes of two places

c) Due to difference in longitudes of two places

d) Due to difference in proximity with ocean of the two places

Solution: b)

Explanation & Learning: The latitudinal extent influences the


duration of the day and night, as one moves from south to north.

At the Equator, the daytime period is always almost twelve hours in


length, no matter what the season. The sun always rises nearly
perpendicular to the horizon, and moves in a similar fashion.

In other latitudes, sun’s rays are more slanted than they are at equator.
So, its natural that days will be shorter in higher latitudes (as evening
and night are caused earlier due to slant of sun’s rays).

As latitude increases to 80° (polar circles - north or south) day length


can be seen to increase to 24 hours or decrease to zero (depending on
time of year).

Hence, option (b).

If the question was about difference in temperatures of day and night


answer would have been (d). If it had been about mean temperatures,
answer would have been (a), (b) and (c).

Q Source: Chapter 1: Contemporary India - I: Page 2

28. Consider the following statements.

1. Lok Sabha exercises more powers in matters of money bills than


Rajya Sabha.


PRELIMS TEST SERIES 2016 MOCK TEST - 1
INSIGHTS

WWW.INSIGHTSONINDIA.COM WWW.INSIGHTSIAS.COM
TEST - 3

2. Council of Ministers can be appointed only by the Lok Sabha.

Which of the above is/are correct?

a) 1 only

b) 2 only

c) Both 1 and 2

d) None

Solution: a)

Explanation & Learning: Once the Lok Sabha passes the budget of
the government or any other money related law, the Rajya Sabha cannot
reject it. The Rajya Sabha can only delay it by 14 days or suggest changes
in it. The Lok Sabha may or may not accept these changes. Thus,
statement 1 is correct.

Council of Ministers is not appointed by the Lok Sabha. It is done by the


President. Ministers can come both from Lok Sabha and Rajya Sabha.
Statement 2 is incorrect.

Q source: Chapter 5: Democratic Politics: Page 84-85

29. Consider the following statements about In-vitro fertilization.

1. It leads to irreversible genetic changes in the embryo.

3. It is illegal in India.

Which of the above is/are correct?

a) 1 only

b) 2 only

c) Both 1 and 2

d) None

Solution: d)


PRELIMS TEST SERIES 2016 MOCK TEST - 1
INSIGHTS

WWW.INSIGHTSONINDIA.COM WWW.INSIGHTSIAS.COM
TEST - 3

Explanation: Though nations like Costa Rica, and China (for some
time) banned IVF, it is not banned in India. In fact, the second successful
IVF birth took place in India. So, statement 2 is wrong.

Even if the test tube baby (IVF) is placed in a surrogate mother, chances
of genetic disorder are very little. Otherwise, genetic changes do not
occur in a test tube baby. So, statement 1 is wrong.

Learning: In vitro fertilization or fertilisation (IVF) is a process by


which an egg is fertilised by sperm outside the body: in vitro ("in glass").
The fertilised egg (zygote) is cultured for 2–6 days in a growth medium
and is then implanted in the same or another woman's uterus, with the
intention of establishing a successful pregnancy.

It is a technique of assisted reproductive technology for treatment of


infertility.

Due to the costs of the procedure, IVF is mostly attempted only after less
expensive options have failed.

Some countries ban or otherwise regulate the availability of IVF


treatment, giving rise to fertility tourism.

Q Source: Chapter 9: Science 8th NCERT: Page 105

30. Even though electromagnetism is far stronger than gravitation,


electrostatic attraction is not relevant for large celestial bodies. Why?

1. Net electric charge of such bodies is zero.

2. Electromagnetic force is a short-range force.

Which of the above is/are correct?

a) 1 only

b) 2 only

c) Both 1 and 2

d) None

Solution: a)


PRELIMS TEST SERIES 2016 MOCK TEST - 1
INSIGHTS

WWW.INSIGHTSONINDIA.COM WWW.INSIGHTSIAS.COM
TEST - 3

Explanation: Electromagnetic attraction is not relevant for large


celestial bodies, such as planets, stars, and galaxies, simply because such
bodies contain equal numbers of protons and electrons and so have a net
electric charge of zero. Nothing "cancels" gravity, since it is only
attractive, unlike electric forces which can be attractive or repulsive. So,
statement 1 is correct.

Electromagnetism is infinite-ranged like gravity. So, statement 2 is


wrong.

Learning: Gravitation is by far the weakest of the four interactions. For


e.g. even a small magnet is able to hold the pin against the gravitational
pull of the entire Earth!

Yet gravitation is very important for macroscopic objects and over


macroscopic distances for the following reasons. Gravitation:

• is the only interaction that acts on all particles having mass, energy
and/or momentum;
• has an infinite range, like electromagnetism but unlike strong and
weak interaction;
• cannot be absorbed, transformed, or shielded against;
• always attracts and never repels.

Q Source: Improvisation: Chapter 11: Science 8th NCERT: Page 137

31. APJ Abdul Kalam Amrut Yojna relates to

a) Providing one cooked meal to pregnant, lactating women in tribal


areas

b) Improving the vaccination rates in rural India to tackle preventable


diseases

c) Registering all Mid-Day meal serving NGOs and providing them single
window administrative clearances

d) None of the above

Solution: a)


PRELIMS TEST SERIES 2016 MOCK TEST - 1
INSIGHTS

WWW.INSIGHTSONINDIA.COM WWW.INSIGHTSIAS.COM
TEST - 3

Explanation & Learning: To curb malnutrition among tribal children


by providing nutrition to pregnant women for 6 months when the child’s
growth is at its peak. It was recently approved by Maharashtra
government.

• The scheme is an initiative of Tribal Development department of


state government.

• Implementation will be done by Anganwadis falling under Women


and Child Welfare Department in 16 districts of states having tribal
population will implement it.

• The scheme Plan is to provide one full hot cooked nutritious food
to every pregnant (in third trimester) and lactating mother (first
three months post-delivery) in tribal areas.

Q Source: http://www.dnaindia.com/india/report-maharashtra-
government-s-free-for-moms-meal-eyes-healthy-tribal-kids-2141766

32. The National River Conservation Directorate (NRCD) is under

a) Ministry of Environment and Forests

b) Ministry of Drinking Water and Sanitation

c) Ministry of Water Resources

d) Ministry of Urban Development

Solution: a)

Explanation & Learning: The National River Conservation


Directorate (NRCD) in the Ministry of Environment, Forests and Climate
Change is implementing the Centrally Sponsored Schemes of National
River Conservation Plan (NRCP) and National Plan for Conservation of
Aquatic Eco-systems’(NPCA) for conservation of rivers, lakes and
wetlands in the country.

The objective of the River Action Plans is to improve water quality of


rivers through implementation of pollution abatement schemes in


PRELIMS TEST SERIES 2016 MOCK TEST - 1
INSIGHTS

WWW.INSIGHTSONINDIA.COM WWW.INSIGHTSIAS.COM
TEST - 3

identified polluted stretches of rivers. NPCA aims at conserving aquatic


ecosystems (lakes and wetlands) through implementation of sustainable
conservation plans, and governed with application of uniform policy and
guidelines.

Q Source: Improvisation: Chapter 3: Contemporary India – I: Page 23


(NRCP box)

33. As compared to their Himalayan counterparts, Peninsular Rivers


have

1. Shorter courses

2. Deeper beds

3. Higher gradient load

Select the correct answer using the codes below.

a) 1 only

b) 2 and 3 only

c) 1 and 3 only

d) All of the above

Solution: a)

Explanation: Many peninsular rivers flow over hard rocky surfaces,


where bed or lateral erosion is not significant, leading to lower sediment
load.

On the other hand, in Himalayan Rivers, course is deeper, gradient


higher and course longer, which results in greater sediment load.

So, only statement 1 is correct, others are wrong.

Learning: Himalayan Rivers have many other depositional features in


their floodplains. They also have well-developed Deltas.

Most of the rivers of peninsular India originate in the Western Ghats and
flow towards the Bay of Bengal.


PRELIMS TEST SERIES 2016 MOCK TEST - 1
INSIGHTS

WWW.INSIGHTSONINDIA.COM WWW.INSIGHTSIAS.COM
TEST - 3

Thus, a large number of the Peninsular Rivers are seasonal, as their flow
is dependent on rainfall. During the dry season, even the large rivers
have reduced flow of water in their channels.

Q Source: Chapter 3: Contemporary India – I: Page 18

34. Which of the following institutions can expand and increase the
scope of fundamental rights in the constitution?

1. President of India

2. Supreme Court

3. High Court

4. Parliament

5. National Human Rights Commission

Select the correct answer using the codes below.

a) 2 and 4 only

b) 2, 3 and 4 only

c) 1, 2, 4 and 5 only

d) 1 and 4 only

Solution: b)

Explanation: In matters of constitutional amendment (e.g. amending


fundamental right), the President is a mere rubber stamp. He cannot
disagree or send back any constitutional amendment. He must sign it.
Statement 1 is this incorrect.

Alongwith SC, HCs also interpret and expand fundamental rights. For
e.g. Delhi HC gave a judgment on LGBT community, by declaring section
377 IPC as unconstitutional. SC has expanded the meaning of Right to
life and liberty (Article 21) which now also covers Right to a safe
environment, Right to Dignity etc – all this without any formal
amendment of the constitution. Statement 2 and 3 are thus correct.


PRELIMS TEST SERIES 2016 MOCK TEST - 1
INSIGHTS

WWW.INSIGHTSONINDIA.COM WWW.INSIGHTSIAS.COM
TEST - 3

Parliament can directly amend the constitution and expand the scope of
fundamental rights. So, statement 4 is also correct.

NHRC does not have such powers. At the most it may recommend the
government some steps to improve the situation of human rights in the
country. So, statement 5 is incorrect.

Q source: Chapter 6: Democratic Politics: Page 109

35. Eyes can be donated by people

1. Of any age

2. Using spectacles

3. Suffering from AIDS

4. Suffering from Tetanus and Cholera

Select the correct answer using the codes below.

a) 1 and 3 only

b) 1, 2 and 4 only

c) 2 and 4 only

d) 1 and 2 only

Solution: d)

Explanation & Learning:

Eyes can be donated by any person to visually challenged persons


suffering from corneal blindness, The person may be male or female of
any age; of any social status; using spectacles; suffering from any normal
disease but not AIDS, Hepatitis B or C, Rabies, Leukemia, Tetanus,
Cholera, Encephalitis.

The eyes have to be donated within 4-6 hours after death at any place,
home or hospital.

A person who wants to donate the eyes has to pledge eyes during his/her
lifetime to any registered eye bank.


PRELIMS TEST SERIES 2016 MOCK TEST - 1
INSIGHTS

WWW.INSIGHTSONINDIA.COM WWW.INSIGHTSIAS.COM
TEST - 3

Q Source: Improvisation: Chapter 16: Science 8th NCERT: Page 216

36. If there is a difference between the two Houses of Parliament on


ordinary legislation, the matter

a) must be referred to the President of India, whose opinion shall be final


and binding

b) can be referred to the Supreme Court of India for its advisory


jurisdiction

c) will be referred to a joint sitting of the Parliament convened by the


President

d) will be settled through a Joint Parliamentary Committee setup by both


the houses

Solution: c)

Explanation & Learning: Any ordinary law needs to be passed by


both the Houses. But if there is a difference between the two Houses, the
final decision is taken in a joint session in which members of both the
Houses sit together. Because of the larger number of members, the view
of the Lok Sabha is likely to prevail in such a meeting.

However, Lok Sabha exercises more powers in money matters. Once the
Lok Sabha passes the budget of the government or any other money
related law, the Rajya Sabha cannot reject it. The Rajya Sabha can only
delay it by 14 days or suggest changes in it. The Lok Sabha may or may
not accept these changes.

Q source: Chapter 5: Democratic Politics: Page 84

37. When the earth crosses the tail of a comet, it indicates

a) an impending solar spot cycle

b) possible sight of a meteor shower


PRELIMS TEST SERIES 2016 MOCK TEST - 1
INSIGHTS

WWW.INSIGHTSONINDIA.COM WWW.INSIGHTSIAS.COM
TEST - 3

c) an impending high tide event in oceans

d) None of the above

Solution: b)

Explanation: A meteor shower is a celestial event in which a number of


meteors are observed to radiate, or originate, from one point in the night
sky.

A meteor shower is the result of an interaction between a planet, such as


Earth, and streams of debris from a comet.

Learning: A comet is an icy small Solar System body that, when passing
close to the Sun, heats up and begins to outgas, displaying a visible
atmosphere or coma, and sometimes also a tail. These phenomena are
due to the effects of solar radiation and the solar wind upon the nucleus
of the comet.

Some people think that comets are messengers of disasters, such as wars,
epidemics and floods. But these are all myths and superstitions.
Appearance of a comet is a natural phenomenon.

Q Source: Chapter 17: Science 8th NCERT: Page 233

38. Which of the following institutions resolves the disputes between


Centre and States arising out of official decisions?

a) Union Finance Commission (UFC)

b) Supreme Court (SC)

c) Central Administrative Tribunal (CAT)

d) Chairman, Rajya Sabha

Solution: b)

Explanation: Finance Commission is not a dispute resolving body. It


recommends allocation of tax resources between Centre and states. So,
option (a) is wrong.


PRELIMS TEST SERIES 2016 MOCK TEST - 1
INSIGHTS

WWW.INSIGHTSONINDIA.COM WWW.INSIGHTSIAS.COM
TEST - 3

CAT resolves disputes between the officials and government with regard
to personnel matters like promotion, recruitment etc. So, option (c) is
wrong.

Chairman, RS, only presides over the meetings of the Rajya Sabha. Rajya
Sabha may discuss disputes over decisions of Centre and States. It
cannot pass a final and binding judgment alone. So, option (d) is wrong.

Supreme Court and the High Courts in India settle disputes arising out
of governmental decisions. For e.g. reservation of creamy layer OBCs in
‘Indira Sawhney and others Vs Union of India case’.

Q source: Chapter 5: Democratic Politics: Page 81

39. Water can be considered polluted due to

1. High concentration of nutrients

2. Higher temperature

3. Presence of larvae eating fish

Select the correct answer using the codes below.

a) 1 and 2 only

b) 2 and 3 only

c) 1 and 3 only

d) None of the above

Solution: a)

Explanation: Freshwater fishes like mosquitofish are used to feed on


larvae to biocontrol mosquito population. Such fishes are not a sign of
water pollution. Statement 3 is incorrect.


PRELIMS TEST SERIES 2016 MOCK TEST - 1
INSIGHTS

WWW.INSIGHTSONINDIA.COM WWW.INSIGHTSIAS.COM
TEST - 3

Hot water released from plants and industries raises the average
temperature of the waterbody, adversely affecting the animals and plants
living in it. It is a form of pollution. Statement 2 is correct.

Statement 1 causes eutrophication (algal boom) in water bodies resulting


in higher consumption of oxygen and loss to aquatic life: hence, correct.

Q Source: Improvisation: Chapter 18: Science 8th NCERT: Page 249

40. The climate and associated weather conditions in India are governed
by

1. Pressure and surface winds

2. Upper air circulation

3. Tropical easterly jet stream

Select the correct answer using the codes below.

a) 1 only

b) 2 and 3 only

c) 3 only

d) All of the above

Solution: d)

Explanation & Learning: Consider this - during winter, there is a


high-pressure area north of the Himalayas. During summer, there is a
low pressure trough in the Northern plains. Winds blow and fill the
depression also bringing monsoon. So, clearly statement 1 is correct.

For statement 2- jet streams are an important component of upper air


circulation which is dominated by westerly flow. These jet streams blow
south of Himalayas, all through the year except in summer affecting
India’s climate and weather. So, statement 2 is also correct.

Also, the western (not eastern) cyclonic disturbances experienced in the


north and north-western parts of the country are brought in by this
westerly flow which affects rainfall in Northern and north-western India.


PRELIMS TEST SERIES 2016 MOCK TEST - 1
INSIGHTS

WWW.INSIGHTSONINDIA.COM WWW.INSIGHTSIAS.COM
TEST - 3

In summer, the subtropical westerly jet stream moves north of the


Himalayas with the apparent movement of the sun. An easterly jet
stream, called the tropical easterly jet stream blows over peninsular
India, approximately over 14°N during the summer months. So,
statement 3 is also correct.

Q Source: Chapter 4: Contemporary India – I: Page 28

41. Which of the following is NOT essential to have and sustain a


democracy?

a) Protection of the fundamental rights of all citizens

b) Freedom to express views, freedom to organise and freedom to protest

c) Political equality of citizens

d) A written constitution

Solution: d)

Explanation: A democracy cannot exist without giving some


fundamental rights of citizens and to safeguard them via institutional
mechanisms. Option (a) is essential.

Without freedom to opine, express, organize and protest a government


can turn tyrannical and authoritarian. It acts as a check and balance.
Hence, option (b) is essential.

Political equality of citizens refers to equal voting rights, equal eligibility


for public offices etc. In a sense, it is one man, one vote as per BR
Ambedkar. Option (c) is the heart of democracy.

A democracy can exist without a written constitution. Britain is the best


example. Moreover, nations having a written constitution may not be


PRELIMS TEST SERIES 2016 MOCK TEST - 1
INSIGHTS

WWW.INSIGHTSONINDIA.COM WWW.INSIGHTSIAS.COM
TEST - 3

democratic, e.g, Syria. However, it is essential to have laws/rules that


protect citizen’s rights. Option (d) is correct.

Q source: Chapter 1: Democratic Politics

42. Sandalwood, khair, kusum, arjun and mulberry which are


commercially important tree species are found in

1. Tropical deciduous forests

2. Tropical evergreen forests

3. Montane forests

Select the correct answer using the codes below.

a) 1 and 2 only

b) 1 only

c) 2 and 3 only

d) 2 only

Solution: b)

Explanation & Learning: Silver fir, junipers, pines and birches are
the common trees of montane forests. So, statement 3 is wrong.

Teak is the most dominant species of deciduous forests. In dry deciduous


forests, Sal, Peepal, Neem grow. In moist deciduous, the ones mentioned
in the question grow. So, statement 1 is correct.

On the other hand, in evergreen forests, commercially important trees


like ebony, mahogany, rosewood, rubber and cinchona grow. So,
statement 2 is wrong.

Q Source: Chapter 5: Contemporary India – I: Page 46

43. Which of the following organizations provide direct loans to


governments?


PRELIMS TEST SERIES 2016 MOCK TEST - 1
INSIGHTS

WWW.INSIGHTSONINDIA.COM WWW.INSIGHTSIAS.COM
TEST - 3

a) International Monetary Fund (IMF)

b) World Bank (WB)

c) Asian Development Bank (ADB)

d) All of the above

Solution: d)

Explanation & Learning: World Bank provides low-interest loans,


zero to low-interest credits, and grants to developing countries. These
support a wide array of investments in such areas as education, health,
public administration, infrastructure, financial and private sector
development, agriculture, and environmental and natural resource
management.

The International Monetary Fund (IMF) is an organization of 188


countries, working to foster global monetary cooperation, secure
financial stability, facilitate international trade, promote high
employment and sustainable economic growth, and reduce poverty
around the world.

ADF from ADB is a major instrument of concessional financing that has


supported equitable and sustainable development in Asia since 1973.
Funded by ADB's member countries, it offers loans at very low interest
rates as well as grants to help reduce poverty in ADB's poorest member
countries.

Q source: Chapter 1: Democratic Politics: Page 15

44. In 2005, the Supreme Court has declared the Presidential


Proclamation dissolving the Bihar Assembly as unconstitutional. Among
the following, which democratic feature of Indian polity does this case
safeguards?

a) Respect for Rights

b) Rule of Law

c) Right to Public office


PRELIMS TEST SERIES 2016 MOCK TEST - 1
INSIGHTS

WWW.INSIGHTSONINDIA.COM WWW.INSIGHTSIAS.COM
TEST - 3

d) Universal Adult Franchise

Solution: b)

Explanation: There is no ‘right to public office’ in a democracy. It is


equality of opportunity in matters of public employment. So, option (c) is
wrong.

There are no fundamental rights involved in adjudicating such cases.


Option (a) is wrong.

Universal adult franchise was not affected in any way by the dissolution
and the order of SC. Option (d) is wrong.

The SC held that dissolution was not as per the provisions of the
constitution and illegal. Hence, safeguarding the assembly was the
protection of rule of law. Option (b) is correct.

Q source: Chapter 2: Democratic Politics: Based on concepts

45. National Green Tribunal (NGT) has recently banned straw burning in
some North-western states of India because it emitted

1. Lead

2. Arsenic

3. Particulate Matter

4. Hexafluroine

Select the correct answer using the codes below.

a) 1 and 2 only

b) 3 only

c) 3 and 4 only

d) All of the above

Solution: b)


PRELIMS TEST SERIES 2016 MOCK TEST - 1
INSIGHTS

WWW.INSIGHTSONINDIA.COM WWW.INSIGHTSIAS.COM
TEST - 3

Explanation & Learning: Straw Burning is age-old agricultural


practice followed by farmers biannually by setting fire to their fields to
clear the land of residue (Straw) of one harvest and sow the next.

It is mainly carried in Northern western states in the month of


November to save time under the rice-wheat crop rotation and to make
the land more fertile and kill pests.

The open field burning emits Carbon Dioxide (CO2), Carbon monoxide
(CO) and Nitrous Oxide (NO) along with particulate matter

Earlier, Union Government had proposed to provide incentives to


farmers and promoting efficient farming practices with subsidies to stop
this menace.

Q Source: http://www.hindustantimes.com/chandigarh/ignoring-ngt-
orders-burning-of-straw-on-in-punjab-haryana/story-
ULq4klXQ4dG494P0wC4MWJ.html

46. Large scale deforestation in a forest can cause fertility of soil to


decline. Why?

1. Soil can be leached easily due to heavy rain

2. Microbial activity in soil increases significantly

3. Nutrient content of the soil changes.

Select the correct answer using the codes below.

a) 1 and 2 only

b) 2 and 3 only

c) 1 and 3 only

d) 1 only

Solution: c)

Explanation: Removal of the top layer of the soil during deforestation


exposes the lower, hard and rocky layers. This soil has less humus and is
less fertile. So, statement 3 is correct.


PRELIMS TEST SERIES 2016 MOCK TEST - 1
INSIGHTS

WWW.INSIGHTSONINDIA.COM WWW.INSIGHTSIAS.COM
TEST - 3

Microbial activity in soil actually decreases due to the loss of biodiversity


and the forest. Statement 2 is incorrect.

As the soil is exposed, rainwater can easily leach washing off nutrients in
run-off water. Statement 1 is correct.

Learning: Deforestation is a major cause which leads to the change in


soil properties.

Physical properties of the soil get affected by plantation and vegetation.


Trees prevent soil erosion. Fewer trees result in more soil erosion.
Gradually the fertile land gets converted into deserts. It is called
desertification.

Deforestation also leads to a decrease in the water holding capacity of the


soil.

The movement of water from the soil surface into the ground
(infiltration rate) is reduced. So, there are floods. The other properties of
the soil like nutrient content, texture, etc., also change because of
deforestation.

Q Source: Chapter 7: Science 8th NCERT: Page 78

47. Consider the following about Antibiotics.

1. They are used against both bacteria and viruses.

2. When taken unnecessarily they may kill the beneficial bacteria in


the body.

3. Regular use of high doses of antibiotics may make them


ineffective.

Select the correct answer using the codes below.

a) 2 only

b) 1 and 3 only

c) 1 and 2 only

d) 2 and 3 only


PRELIMS TEST SERIES 2016 MOCK TEST - 1
INSIGHTS

WWW.INSIGHTSONINDIA.COM WWW.INSIGHTSIAS.COM
TEST - 3

Solution: d)

Explanation: Antibiotics do not work against viruses. Many common


respiratory tract infections (RTIs) — such as colds and flu — are caused
by viruses, where they can’t be of any use. So, statement 1 is incorrect.

Frequent and high usage of anti-biotics makes the patient suffer from
antibiotic resistance where further strains of bacteria are unaffected by
the antibiotic. Besides, antibiotics kill bacteria without much
discrimination. Beneficial bacteria may also be killed. So, statements 2
and 3 are correct.

Learning: They are a type of antimicrobial used in the treatment and


prevention of bacterial infection. They may either kill or inhibit the
growth of bacteria. Several antibiotics are also effective against fungi and
protozoans, and some are toxic to humans and animals, even when given
in therapeutic dosage.

Q Source: Chapter 2: Science 8th NCERT: Page 21

48. Main tributaries of Kaveri are

1. Bhavani

2. Amravati

3. Brahmani

4. Baitarini

Select the correct answer using the codes below.

a) 1 and 2 only

b) 2 and 3 only

c) 3 and 4 only

d) 1 and 4 only

Solution: a)


PRELIMS TEST SERIES 2016 MOCK TEST - 1
INSIGHTS

WWW.INSIGHTSONINDIA.COM WWW.INSIGHTSIAS.COM
TEST - 3

Learning: The Kaveri rises in the Brahmagri range of the Western


Ghats and it reaches the Bay of Bengal in south of Cuddalore, in Tamil
Nadu. Total length of the river is about 760 km.

Its main tributaries are Amravati, Bhavani, Hemavati and Kabini. Its
basin drains parts of Karnataka, Kerala and Tamil Nadu.

The river Kaveri makes the second biggest waterfall in India. It is known
as Sivasamudram.

The Damoder, the Brahmani, the Baitarni and the Subarn rekha are
major rivers flowing towards east, not tributaries.

Q Source: Chapter 3: Contemporary India – I: Page 22

49. Consider the following about the Election Commission of India


(ECI).

1. The Chief Election Commissioner (CEC) is appointed by the


President of India.

2. It enjoys the same kind of independence that the judiciary


enjoys.

3. Once appointed, the Chief Election Commissioner is fully


answerable to the government for its powers and responsibilities.

Select the correct answer using the codes below.

a) 1 only

b) 2 and 3 only

c) 1 and 3 only

d) 1 and 2 only

Solution: d)

Explanation: The ECI is an independent body. It is not answerable to


the government. Answerability on day to day matters and even major


PRELIMS TEST SERIES 2016 MOCK TEST - 1
INSIGHTS

WWW.INSIGHTSONINDIA.COM WWW.INSIGHTSIAS.COM
TEST - 3

policy decisions can tend to erode autonomy. Its autonomy is central to


the Indian democracy. Hence, statement 3 is incorrect.

ECI commissioners enjoy security of tenure, salary charged on


Consolidated Fund of India, independence from government
interference, difficult process of impeachment etc. Hence, they enjoy
similar independence as judges do.

Q source: Improvisation: Chapter 4: Democratic Politics: Page 69

50. Why are the western slopes of the Western Ghats covered with thick
forests and not the eastern slopes?

1. Eastern side falls in rain-shadow zone

2. No protected forests exist on the eastern slopes.

3. Soil in eastern slope cannot support vegetation due to absence of


nutrients.

Select the correct answer using the codes below.

a) 1 and 2 only

b) 2 and 3 only

c) 1 only

d) 1 and 3 only

Solution: c)

Explanation & Learning: When moisture laden winds cause rainfall


in western slopes, upon moving to eastern sides they lose much of their
moisture. Rainfall is less, and thus vegetation is less thick at eastern
slopes. So, statement 1 is correct.

Nilgiri Biosphere Reserve extends to Eastern Slopes as well. So,


statement 2 is wrong.


PRELIMS TEST SERIES 2016 MOCK TEST - 1
INSIGHTS

WWW.INSIGHTSONINDIA.COM WWW.INSIGHTSIAS.COM
TEST - 3

Eastern slopes does support vegetation, as the soil is not highly leached.
It contains red as well as laterite soil. Statement 3 is incorrect.

Q Source: Improvisation: Chapter 5: Contemporary India – I: Page 43

51. Consider the following about blue-green algae.

1. They can fix atmospheric nitrogen in aerobic conditions.

2. They can be used as bio-fertilizer.

3. They do not perform photosynthesis unlike normal algae.

4. They can be used to produce algal biofuel.

Select the correct answer using the codes below.

a) 1, 2 and 4 only

b) 2 and 4 only

c) 2 and 3 only

d) All of the above

Solution: a)

Explanation & Learning: Blue-green Algae, also called as


Cyanobacteria obtain their energy through photosynthesis.

Nitrogen-fixation by the cyanobacteria such as (Anabaena, a symbiont of


the aquatic fern Azolla), provides rice plantations with biofertilizer. Free-
living cyanobacteria are also present in the water column in rice paddies

Recent studies suggest that significant exposure to high levels of some


species of cyanobacteria producing toxins such as BMAA can cause
amyotrophic lateral sclerosis (ALS). The Gulf War veterans' cluster is a
notable example.

Currently, efforts are underway to commercialize algae-based fuels such


as diesel, gasoline, and jet fuel. Like fossil fuel, algae fuel releases CO2
when burnt, but unlike fossil fuel, algae fuel and other biofuels only


PRELIMS TEST SERIES 2016 MOCK TEST - 1
INSIGHTS

WWW.INSIGHTSONINDIA.COM WWW.INSIGHTSIAS.COM
TEST - 3

release CO2 recently removed from the atmosphere via photosynthesis


as the algae or plant grew.

Recent research has suggested the potential application of cyanobacteria


to the generation of renewable energy by converting sunlight into
electricity.

Several cases of human poisoning have also been documented due to


blue-green algae, but a lack of knowledge prevents an accurate
assessment of the risks.

Q Source: Improvisation: Chapter 2: Science 8th NCERT: Page 22

52. The highest formal authority in India is carried by

a) Prime Minister of India

b) President of India

c) Cabinet Secretary

d) Heads of independent Constitutional bodies

Solution: b)

Explanation: President is the head of the state and is the highest


formal authority in the country. All executive actions of the government
are taken in his name.

Prime Minister is the head of the government and actually exercises all
governmental powers. He takes most of the decisions in the Cabinet
meetings. So, option (b) is correct.

Learning: As per Article 53 of the Constitution-


PRELIMS TEST SERIES 2016 MOCK TEST - 1
INSIGHTS

WWW.INSIGHTSONINDIA.COM WWW.INSIGHTSIAS.COM
TEST - 3

(a) The executive power of the Union shall be vested in the President
and shall be exercised by him either directly or through officers
subordinate to him in accordance with this Constitution.

(b) Without prejudice to the generality of the foregoing


provision, the supreme command of the Defence Forces of the
Union shall be vested in the President and the exercise thereof
shall be regulated by law.

Q source: Chapter 5: Democratic Politics: Page 80

53. Which of the following planets in the solar system are less dense than
water?

a) Mercury, Venus and Saturn

b) Venus and Neptune

c) Mars and Jupiter

d) Saturn only

Solution: d)

Learning: Saturn has the lowest density of all the planets in the Solar
System. The actual number is 0.687 grams per cubic centimeter. This is
actually less dense than water; if you had a large enough pool of water,
Saturn would float.

Jupiter has an average density of 1.33 grams per cubic centimeter. So it


wouldn’t float on water. And Earth, the densest planet in the Solar
System, measures 5.51 grams/cubic centimeter.

Q Source: Chapter 17: Science 8th NCERT: Page 231

54. The Council of Ministers is held accountable to and controlled by

a) Both houses of Parliament


PRELIMS TEST SERIES 2016 MOCK TEST - 1
INSIGHTS

WWW.INSIGHTSONINDIA.COM WWW.INSIGHTSIAS.COM
TEST - 3

b) Chairman, Rajya Sabha

c) Lok Sabha

d) President of India

Solution: c)

Explanation: Parliament consists of both houses. Rajya Sabha is not an


elected chamber and thus cannot control the council of ministers. Option
(a) is wrong.

Hence, option (b) is also wrong.

President of India can only advise/question the CoM on matters sent by


it for consideration. Option (d) is wrong.

Learning: Only a person who enjoys the support of the majority of the
members in the Lok Sabha is appointed the Prime Minister. If the
majority of the Lok Sabha members say they have ‘no confidence’ in the
Council of Ministers, all ministers including the Prime Minister, have to
quit. The Rajya Sabha does not have this power.

Q source: Chapter 5: Democratic Politics

55. Carbon monoxide can be produced from or found in

1. Incomplete burning of petrol and diesel

2. Naturally in groundwater and ice caps

3. Operation of a kitchen stove in enclosed space

4. Eruption of volcanoes

Select the correct answer using the codes below.

a) 1 and 3 only

b) 2 and 4 only

c) 1, 2 and 3 only

d) 1, 3 and 4 only


PRELIMS TEST SERIES 2016 MOCK TEST - 1
INSIGHTS

WWW.INSIGHTSONINDIA.COM WWW.INSIGHTSIAS.COM
TEST - 3

Solution: d)

Explanation: Any hydrocarbon fuel when oxidized in low presence of


oxygen, it forms carbon monoxide instead of dioxide. So, statements 1
and 3 are correct.

Carbon dioxide (not carbon monoxide) is found naturally in


groundwater, ice caps, glaciers etc. So, statement 2 is incorrect.

The principal components of volcanic gases are water vapour (H2O),


CO2, CO, SO2 or hydrogen sulphide, nitrogen, argon, helium, neon,
methane and hydrogen. So, statement 4 is correct.

Learning: It is also generated from natural and man-made fires (such


as forest and bushfires, burning of crop residues, and sugarcane fire-
cleaning). Carbon monoxide occurs dissolved in molten volcanic rock at
high pressures in the Earth's mantle.

Worldwide, the largest source of carbon monoxide is natural in origin,


due to photochemical reactions in the troposphere.

Q Source: Improvisation: Chapter 18: Science 8th NCERT: Page 242

56. The last natural refuge of the endangered Manipur brow-antlered


deer (Sangai) is in

a) Loktak Lake

b) Hajong Lake

c) Sendra Lake

d) Kolleru Lake

Solution: a)

Learning: The sangai is an endemic, rare and endangered subspecies of


brow-antlered deer found only in Manipur, India. It is also the state
animal of Manipur.

Its habitat is located in the southern parts of the Loktak Lake, which is
the largest freshwater lake in eastern India. It is also one of the seven


PRELIMS TEST SERIES 2016 MOCK TEST - 1
INSIGHTS

WWW.INSIGHTSONINDIA.COM WWW.INSIGHTSIAS.COM
TEST - 3

Ramsar sites of international importance. The habitat of the sangai is


now protected as the Keibul Lamjao National Park.

The sangai was believed to be almost extinct by 1950.

Habitat destruction and poaching are two major threats to sangai.

Q Source: Improvisation: Chapter 3: Contemporary India – I: Page 22


(image of Loktak lake)

57. Which of the following statements about the provisions of the Forest
Conservation Act (1980) is CORRECT?

a) State governments cannot de-reserve forests without the prior


approval of Central Government.

b) It consolidates the law relating to forests, the transit of forest-


produce.

c) It recognizes the rights of traditional forest dwellers over the forest


areas.

d) National parks and biosphere reserves are constituted under this Act.

Solution: a)

Explanation: National parks and biosphere reserves are constituted


under Wildlife Conservation Act. So, option (d) is wrong.

The Indian Forest Act, 1927 consolidates the law relating to forests, the
transit of forest-produce and the duty leviable on timber and other
forest-produce. So, option (b) is wrong.

The Scheduled Tribes and Other Traditional Forest Dwellers


(Recognition of Forest Rights) Act, 2006, recognizes the rights of forest-
dwelling Scheduled Tribes and other traditional forest dwellers over the
forest areas inhabited by them and provides a framework for according
the same. So, option (c) is wrong.

Learning: The Forest Conservation Act 1980 was enacted to help


conserve the country's forests. It strictly restricts and regulates the de-
reservation of forests or use of forest land for non-forest purposes


PRELIMS TEST SERIES 2016 MOCK TEST - 1
INSIGHTS

WWW.INSIGHTSONINDIA.COM WWW.INSIGHTSIAS.COM
TEST - 3

without the prior approval of Central Government. To this end the Act
lays down the pre-requisites for the diversion of forest land for non-
forest purposes.

Q Source: Improvisation: Chapter 7: Science 8th NCERT: Page 85

58. Which of the following rivers do NOT make ‘delta’?

1. Mahanadi

2. Tapi

3. Narmada

4. Godavari

Select the correct answer using the codes below.

a) 1 and 2 only

b) 2 and 3 only

c) 3 and 4 only

d) 1 and 4 only

Solution: b)

Explanation & Learning: Most of the major rivers of the Peninsula


such as the Mahanadi, the Godavari, the Krishna and the Kaveri flow
eastwards and drain into the Bay of Bengal. These rivers make deltas at
their mouths. There are numerous small streams flowing west of the
Western Ghats.

The Narmada and the Tapi are the only long rivers, which flow west and
make estuaries. The drainage basins of the peninsular rivers are
comparatively small in size.

Q Source: Chapter 3: Contemporary India – I: Page 21

58. Right to Constitutional Remedies is


PRELIMS TEST SERIES 2016 MOCK TEST - 1
INSIGHTS

WWW.INSIGHTSONINDIA.COM WWW.INSIGHTSIAS.COM
TEST - 3

a) A fundamental right

b) A constitutional right

c) A Legal right

d) A right based on the directions issued by the executive

Solution: a)

Explanation: Right to constitutional remedies is a fundamental right.


The Courts can issue writs to enforce this right when any of the
fundamental rights are violated.

You can understand it better this way. Right to vote is not a fundamental
right, but it is a constitutional right. The Supreme Court cannot issue a
writ to enforce a constitutional right. A party can, however, file a case
citing his rights. So, option (a) is correct, not (b).

Legal right is any right conferred by law, for e.g. right to work for 100
days for a household in rural areas is given by MGNREGA.

Learning: Fundamental Rights are guaranteed against the actions of


the Legislatures, the Executive, and any other authorities instituted by
the government. There can be no law or action that violates the
Fundamental Rights. If any act of the Legislature or the Executive takes
away or limits any of the Fundamental Rights it will be invalid.

Q source: Chapter 6: Democratic Politics: Page 107

59. Consider the following about the Indian Agricultural Research


Institute (IARI).

1. It has been established under an Act of the Parliament.

2. It did not contribute to the Indian "Green Revolution" of the


1970s as it was established after it.

Which of the above is/are correct?


PRELIMS TEST SERIES 2016 MOCK TEST - 1
INSIGHTS

WWW.INSIGHTSONINDIA.COM WWW.INSIGHTSIAS.COM
TEST - 3

a) 1 only

b) 2 only

c) Both 1 and 2

d) None

Solution: d)

Explanation: It was established in 1905 at Pusa (Bihar) with a


generous grant from an American philanthropist. No Parliamentary act
established it. So, statement 1 is wrong.

During the fifties, the advancement of scientific disciplines constituted


the core program of IARI and provided the base for its fast expansion in
the 1960’s and 1970’s. So, it also claims credit for enabling the Green
Revolution, in part by developing rust resistant strains of wheat, apart
from the HYVs introduced from abroad.

Learning: Green Revolution in India was a period during which


agriculture in India increased its yields due to improved agronomic
technology.

The introduction of high-yielding varieties of seeds and the increased use


of chemical fertilizers and irrigation led to the increase in production
needed to make India self-sufficient in food grains, thus improving
agriculture in India.

In many areas, Green Revolution is associated with the loss of soil


fertility due to increased use of chemical fertilizers. Also, continuous use
of groundwater for tubewell irrigation has reduced the water -table
below the ground.

Q Source: Improvisation: Chapter 1: Economics 9th NCERT: Page 4

60. Consider the following about National Human Rights Commission.

1. The Commission is independent of the government.


PRELIMS TEST SERIES 2016 MOCK TEST - 1
INSIGHTS

WWW.INSIGHTSONINDIA.COM WWW.INSIGHTSIAS.COM
TEST - 3

2. It is a constitutional authority.

3. Its recommendations are binding on the courts but not


government.

4. There is no fee to approach the NHRC.

Select the correct answer using the codes below.

a) 1 only

b) 1 and 4 only

c) 1, 3 and 4 only

d) 2 and 3 only

Solution: b)

Explanation: The NHRC cannot by itself punish the guilty. That is the
responsibility of courts. The NHRC is there to make independent and
credible inquiry into any case of violation of human rights. Thus, its
recommendations are not binding on neither the courts not the
government. Statement 3 is incorrect.

The commission has been established by law as autonomous of the


government, and not by the constitution, so statement 2 is wrong, and 1
is correct.

Any citizen of India can write a letter to the NHRC to complain against
the violation of human rights. There is no fee or any formal procedure to
approach the NHRC. So, statement 4 is correct.

Q source: Chapter 6: Democratic Politics: Page 108

61. The shape of a motor vehicle affects its

1. Fuel efficiency

2. Speed

3. Overall performance


PRELIMS TEST SERIES 2016 MOCK TEST - 1
INSIGHTS

WWW.INSIGHTSONINDIA.COM WWW.INSIGHTSIAS.COM
TEST - 3

Select the correct answer using the codes below.

a) 1 and 2 only

b) 2 and 3 only

c) 2 only

d) All of the above

Solution: d)

Explanation & Learning: When objects move through fluids (e.g.


air), they have to overcome friction acting on them. Their bodies must be
designed in shapes which would make them not to lose much energy in
overcoming friction. A body that is poorly designed will lose a lot of fuel
in just overcoming the drag exerted on it. Its fuel efficiency will be lesser.
So, statement 1 is correct.

Since drag exerts a backward force, it reduces the speed of the vehicle.
So, statement 2 is correct.

The above also affect a vehicle’s acceleration, wear and tear etc, and thus
affect the vehicle’s overall performance. So, statement 3 is also correct.

Q Source: Improvisation: Chapter 11: Science 8th NCERT: Page 154

62. Which of the following problem(s) occur when using dry ice as a fire
extinguisher?

1. It causes harm electrical equipment nearby resulting in fire.

2. It aggravates fire situations caused due to oil spills.

Which of the above is/are correct?

a) 1 only

b) 2 only

c) Both 1 and 2


PRELIMS TEST SERIES 2016 MOCK TEST - 1
INSIGHTS

WWW.INSIGHTSONINDIA.COM WWW.INSIGHTSIAS.COM
TEST - 3

d) None

Solution: d)

Explanation & Learning: The above problems are not related with
dry ice, but water.

If electrical equipment is on fire, water may conduct electricity and harm


those trying to douse the fire. Water is also not suitable for fires
involving oil and petrol. So, it sinks below the oil, and oil keeps burning
on top.

On the other hand, CO2, being heavier than oxygen, covers the fire like a
blanket. Since the contact between the fuel and oxygen is cut off, the fire
is controlled. The added advantage of CO2 is that in most cases it does
not harm the electrical equipment.

Hence, both statements are incorrect.

Q Source: Chapter 6: Science 8th NCERT: Page 69

63. Red Data Book, IUCN, keeps a record of all the endangered animals
and plants. Which of the following institutions contribute to the red list?

1. BirdLife International

2. World Conservation Monitoring Centre

3. Greenpeace

4. Environmental Defense Fund

Select the correct answer using the codes below.

a) 3 and 4 only

b) 1 and 2 only

c) 1, 3 and 4 only

d) 2, 3 and 4 only


PRELIMS TEST SERIES 2016 MOCK TEST - 1
INSIGHTS

WWW.INSIGHTSONINDIA.COM WWW.INSIGHTSIAS.COM
TEST - 3

Solution: b)

Explanation: You can apply elimination by just knowing that


Greenpeace cannot contribute to a Red List. It only focuses its
campaigning on worldwide issues such as climate change, deforestation.

Moreover, in Test-2 we had explained how Birdlife International


contributes to the threatend Bird List.

So, if statement 3 is wrong and 1 is correct, only option (b) can be the
answer.

Learning: The IUCN Red List of Threatened Species (also known as the
IUCN Red List or Red Data List), founded in 1964, is the world's most
comprehensive inventory of the global conservation status of biological
species. The International Union for the Conservation of Nature (IUCN)
is the world's main authority on the conservation status of species

Major species assessors include BirdLife International, the Institute of


Zoology (the research division of the Zoological Society of London), the
World Conservation Monitoring Centre, and many Specialist Groups
within the IUCN Species Survival Commission (SSC). Collectively,
assessments by these organizations and groups account for nearly half
the species on the Red List.

Q Source: Improvisation: Chapter 7: Science 8th NCERT: Page 79

64. Suez Canal allows ships to travel between

a) North and South America

b) Europe and Russia

c) Europe and South Asia

d) East Africa to South-East Asia

Solution: c)

Explanation & Learning: It is an artificial sea-level waterway in


Egypt, connecting the Mediterranean Sea to the Red Sea.


PRELIMS TEST SERIES 2016 MOCK TEST - 1
INSIGHTS

WWW.INSIGHTSONINDIA.COM WWW.INSIGHTSIAS.COM
TEST - 3

It was opened in 1869. It allows ships to travel between Europe and


South Asia without navigating around Africa thereby reducing the sea
voyage distance between Europe and India by about 7,000 kilometres.

The canal is owned and maintained by the Suez Canal Authority (SCA) of
Egypt. Under the Convention of Constantinople, it may be used "in time
of war as in time of peace, by every vessel of commerce or of war, without
distinction of flag”.

Q Source: Chapter 1: Contemporary India - I: Page 2

65. What describes ‘geosynclines’?

a) A linear break in rocks of the earth’s crust along which frequent


displacement occurs

b) A mass of sand and soil that moves slowly under the influence of
gravity along a confined course

c) An area of relatively low atmospheric pressure, which is found mainly


in temperate regions

d) A narrow, shallow, elongated basin with a sinking bottom where rivers


deposited sediments

Solution: d)

Explanation: Option (a) is called ‘faults’.

Option (b) does not exist. Option (c) is called an area of depression.

Geosyncline is basically a narrow, shallow, elongated basin with a


sinking bottom in which a considerable thickness of sediments was
deposited by the rivers coming from Angara and Gondwanaland. So,
option (d) is correct.

It can be subsequently compressed, deformed, and uplifted into a


mountain range or can become a volcano.

Q Source: Glossary: Contemporary India - I


PRELIMS TEST SERIES 2016 MOCK TEST - 1
INSIGHTS

WWW.INSIGHTSONINDIA.COM WWW.INSIGHTSIAS.COM
TEST - 3

66. Decennial Census exercises in India are conducted and administered


under?

a) Ministry of Planning

b) Ministry of Statistics and Programme Implementation

c) Ministry of Home Affairs

d) Ministry of Social Justice and Empowerment

Solution: c)

Explanation & Learning: The responsibility of conducting the


decennial Census rests with the Office of the Registrar General and
Census Commissioner, India under Ministry of Home Affairs,
Government of India.

The Census Act was enacted in 1948 to provide for the scheme of
conducting population census with duties and responsibilities of census
officers.

Later, the office was also entrusted with the responsibility of


implementation of Registration of Births and Deaths Act, 1969 in the
country.

The Indian Census is the most credible source of information on


Demography (Population characteristics), Economic Activity, Literacy
and Education, Housing & Household Amenities, Urbanisation, Fertility
and Mortality.

Q Source: Improvisation: Chapter 6: Contemporary India – I: Page 53

67. Exotic plants of an area refer to

a) Endemic or indigenous vegetation of that area

b) Vegetation that is not native and has come from outside

c) Naturally growing vegetation which has been left undisturbed by


humans for a long time

d) Vegetation that grows in a specific season after a long time


PRELIMS TEST SERIES 2016 MOCK TEST - 1
INSIGHTS

WWW.INSIGHTSONINDIA.COM WWW.INSIGHTSIAS.COM
TEST - 3

Solution: b)

Explanation: Option (a) refers to endemic plants, for e.g. Wild mango
is endemic to India.

Option (c) refers to Virgin vegetation.

Learning: An introduced, alien, exotic, non-indigenous, or non-native


species is a species living outside its native distributional range, which
has arrived there by human activity, either deliberate or accidental.

• Non-native species can have various effects on the local ecosystem.

• Introduced species that become established and spread beyond the


place of introduction are called invasive species.

• Some have a negative effect on a local ecosystem (e.g. affecting the


food chain of the area, reducing biodiversity, competing for food
and light etc).

• Some introduced species may have no negative effect or only minor


impact.

• Some species have been introduced intentionally to combat pests.


They are called biocontrols and may be regarded as beneficial as an
alternative to pesticides in agriculture.

Q Source: Improvisation: Chapter 5: Contemporary India – I: Page 42

68. When on election duty, government officers work under the control
of

a) The concerned District Administration

b) State Planning Board

c) Chief Secretary of the State

d) Election Commission

Solution: d)


PRELIMS TEST SERIES 2016 MOCK TEST - 1
INSIGHTS

WWW.INSIGHTSONINDIA.COM WWW.INSIGHTSIAS.COM
TEST - 3

Explanation: They work under the control of the Election Commission


of India (or State Election Commission as the case may be). Hence,
option (d)

Learning: It is very common for the Election Commission to reprimand


the government and administration for their lapses. This is because of
the independence vested in the commission and the election officials.

When election officials come to the opinion that polling was not fair in
some booths or even an entire constituency, they order a repoll. The
ruling parties generally comply with the orders of the ECI.

Q source: Chapter 4: Democratic Politics: Page 69

69. Which of these states have more than 50% of land under forest
cover?

1. Himachal Pradesh

2. Odisha

3. Sikkim

4. Karnataka

Select the correct answer using the codes below.

a) 2, 3 and 4 only

b) 1, 2 and 3 only

c) 1 and 4 only

d) 1 and 3 only

Solution: d)

Explanation & Learning: Refer to this link to know state-wise forest


cover. Just see the trends e.g. in North-east.

http://www.iasri.res.in/agridata/08data%5Cchapter1%5Cdb2008tb1_41
.pdf

Q Source: Chapter 5: Contemporary India – I: Page 44


PRELIMS TEST SERIES 2016 MOCK TEST - 1
INSIGHTS

WWW.INSIGHTSONINDIA.COM WWW.INSIGHTSIAS.COM
TEST - 3

70. Which United Nations Organ is charged with the maintenance of


international peace and security as well as recommending admission of
new members to the United Nations (UN)?

a) UN Secretariat

b) UN Trusteeship Council

c) International Court of Justice

d) UN Security Council

Solution: d)

Explanation & Learning: Under the Charter, the Security Council has
primary responsibility for the maintenance of international peace and
security. It has 15 Members, and each Member has one vote. Under the
Charter, all Member States are obligated to comply with Council
decisions.

The Security Council takes the lead in determining the existence of a


threat to the peace or act of aggression.

It calls upon the parties to a dispute to settle it by peaceful means and


recommends methods of adjustment or terms of settlement. In some
cases, the Security Council can resort to imposing sanctions or even
authorize the use of force to maintain or restore international peace and
security.

The Security Council also recommends to the General Assembly the


appointment of the Secretary-General and the admission of new
Members to the United Nations. And, together with the General
Assembly, it elects the judges of the International Court of Justice.

Q source: Chapter 1: Democratic Politics: Page 15

71. The term ‘Green Tax’ refers to

1. Excise taxes on environmental pollutants


PRELIMS TEST SERIES 2016 MOCK TEST - 1
INSIGHTS

WWW.INSIGHTSONINDIA.COM WWW.INSIGHTSIAS.COM
TEST - 3

2. Taxes on goods whose use produces such pollutants

Which of the above is/are correct?

a) 1 only

b) 2 only

c) Both 1 and 2

d) None

Solution: c)

Explanation & Learning: Ecotax (short for Ecological taxation or


Green taxation) refers to taxes intended to promote environmentally
friendly activities via economic incentives.

Recently, in a bid to curb health hazards originating from bursting of


firecrackers, the Pune Bench of the National Green Tribunal has directed
civic bodies to levy Rs. 3,000 as ‘green tax’ from sellers.

• The corpus collected from the tax will be used to clean solid waste
generated from firecrackers at public places.

• Part of the money will be used exclusively for environmental


activities such as planting trees and constructing toilets for women.

Q Source: http://timesofindia.indiatimes.com/City/Pune/Plea-in-
National-Green-Tribunal-seeks-ban-on-
firecrackers/articleshow/44796735.cms

72. How does ‘relief’ play a role in determining the climate of a place?

1. By blocking or allowing wind streams

2. By increasing continentality

Which of the above is/are correct?

a) 1 only

b) 2 only


PRELIMS TEST SERIES 2016 MOCK TEST - 1
INSIGHTS

WWW.INSIGHTSONINDIA.COM WWW.INSIGHTSIAS.COM
TEST - 3

c) Both 1 and 2

d) None

Solution: a)

Explanation & Learning: Continentality refers to the impact on land


climate as the distance from sea increases. The farther a place is from
sea, more likelier is it to experience harsher and extreme climate
conditions. It is not related to relief of a place. Statement 2 is incorrect.

High mountains act as barriers for cold or hot winds; they may also
cause precipitation if they are high enough and lie in the path of rain-
bearing winds. The leeward side of mountains remains dry.

Q Source: Chapter 4: Contemporary India – I: Page 27

73. Which of the following is NOT a greenhouse gas?

a) Nitrogen

b) Ozone

c) Methane

d) Water vapour

Solution: a)

Explanation: Nitrous oxides are greenhouse gases, not nitrogen itself.


Similarly, other major atmospheric constituents, like oxygen and argon
(Ar), are not greenhouse gases. Option (a) is correct.

Learning: The most abundant greenhouse gases in Earth's atmosphere


are: Water vapor , Carbon dioxide, Methane, Nitrous oxide, Ozone,
Chlorofluorocarbons (CFCs).


PRELIMS TEST SERIES 2016 MOCK TEST - 1
INSIGHTS

WWW.INSIGHTSONINDIA.COM WWW.INSIGHTSIAS.COM
TEST - 3

The contribution of each gas to the greenhouse effect is affected by the


characteristics of that gas, its abundance, and any indirect effects it may
cause.

For e.g., the direct radiative effect of a mass of methane is about 72 times
stronger than the same mass of carbon dioxide over a 20-year time
frame but it is present in much smaller concentrations so that its total
direct radiative effect is smaller, in part due to its shorter atmospheric
lifetime.

On the other hand, in addition to its direct radiative impact, methane


has a large, indirect radiative effect because it contributes to ozone
formation.

Q Source: Chapter 18: Science 8th NCERT

74. Westerly cyclonic disturbances affect the climate in

a) Southern India

b) North and north-Western India

c) Eastern India

d) South-eastern India

Solution: b)

Explanation & Learning: The western cyclonic disturbances are


weather phenomena of the winter months brought in by the westerly
flow from the Mediterranean region.

They usually influence the weather of the north and north-western


regions of India. Tropical cyclones occur during the monsoon as well as
in October - November, and are part of the easterly flow. These
disturbances affect the coastal regions of the country.

Q Source: Chapter 4: Contemporary India – I: Page 28


PRELIMS TEST SERIES 2016 MOCK TEST - 1
INSIGHTS

WWW.INSIGHTSONINDIA.COM WWW.INSIGHTSIAS.COM
TEST - 3

75. ‘UDAY’ scheme launched recently by the Union Cabinet relates to

a) Consolidation and revamping of Prime Minister’s Fifteen Point


Programme

b) Financial restructuring of debt of power distribution companies

c) Generating employment in left-wing extremism affected areas

d) Making all government offices energy efficient

Solution: b)

Learning: Ujwal DISCOM Assurance Yojna (UDAY) for financial


restructuring of debt of power distribution companies.

The Scheme aims for financial turnaround and revival of Power


Distribution companies (DISCOMs) and also ensures a sustainable
permanent solution to the problem.

• It allows power DISCOMs in selected states to convert their debt


into state bonds as well as roll out number of measures to improve
efficiency at power plants.

• It seeks to ensure that struggling DISCOMs can shake off years of


losses and start on a path to profitability.

• Assures the rise of vibrant and efficient DISCOMs: through four


initiatives (i) Improve operational efficiencies of DISCOMs (ii)
Reduce of cost of power (iii) Reduce interest cost of DISCOMs (iv)
Enforce financial discipline on DISCOMs through alignment with
State finances.

• It shifts 75 per cent of power DISCOMs debt burden to states’


balance sheets. This step would result in interest cost savings to the
tune of 3-5 per cent.

Q Source: http://pmindia.gov.in/en/news_updates/uday-ujwal-
discom-assurance-yojana-for-financial-turnaround-of-power-
distribution-companies/


PRELIMS TEST SERIES 2016 MOCK TEST - 1
INSIGHTS

WWW.INSIGHTSONINDIA.COM WWW.INSIGHTSIAS.COM
TEST - 3

76. Inter Tropical Convergence Zone is

a) A zone from where all tropical cyclones originate

b) A Zone from where trade winds emerge and travel towards poles

c) A broad trough of low pressure in equatorial latitudes

d) A zone of high pressure where tropical cold currents meet

Solution: c)

Explanation & Learning: The Inter Tropical Convergence Zone


(ITCZ,) is a broad trough of low pressure in equatorial latitudes.

This is where the northeast and the southeast trade winds converge.

This convergence zone lies more or less parallel to the equator but moves
north or south with the apparent movement of the sun.

Q Source: Chapter 4: Contemporary India – I: Page 30

77. Consider the following statements.

1. It rises in Tibet.

2. It enters India in Ladakh.

3. Shyok, Nubra and Hunza are amongst its tributaries.

The above may refer to which river system?

a) Indus

b) Brahmaputra

c) Luni

d) Gandak

Solution: a)

Explanation & Learning: The major Himalayan rivers are the Indus,
the Ganga and the Brahmaputra.


PRELIMS TEST SERIES 2016 MOCK TEST - 1
INSIGHTS

WWW.INSIGHTSONINDIA.COM WWW.INSIGHTSIAS.COM
TEST - 3

The river Indus rises in Tibet, near Lake Mansarowar. Flowing west, it
enters India in the Ladakh district of Jammu and Kashmir. It forms a
picturesque gorge in this part. Several tributaries, the Zaskar, the Nubra,
the Shyok and the Hunza, join it in the Kashmir region.

The Indus flows through Baltistan and Gilgit and emerges from the
mountains at Attock. The Satluj, the Beas, the Ravi, the Chenab and the
Jhelum join together to enter the Indus near Mithankot in Pakistan.

Q Source: Chapter 3: Contemporary India – I: Page 18

78. In a water vessel, total pressure exerted at the bottom of a container


depends on

1. Height at which the container is placed above Mean Sea Level


(MSL)

2. Height of water column

3. Width of water column

Select the correct answer using the codes below.

a) 1 and 2 only

b) 2 and 3 only

c) 1 only

d) 2 only

Solution: a)

Explanation & Learning: The pressure at a given depth in a static


liquid is a result the weight of the liquid acting on a unit area at that
depth plus any pressure acting on the surface of the liquid.

P= Patm (atmospheric pressure)+ dgh (pressure due to water)

D- Density of water; G- Acceleration due to gravity; H- height of water


column


PRELIMS TEST SERIES 2016 MOCK TEST - 1
INSIGHTS

WWW.INSIGHTSONINDIA.COM WWW.INSIGHTSIAS.COM
TEST - 3

As altitude increases, the atmospheric pressure decreases, and the total


pressure as a result also falls.

Also, the pressure due to the liquid alone (not total) at a given depth
depends only upon the density of the liquid ρ and the distance below the
surface of the liquid h.

The pressure at a given depth is independent of direction - it is the same


in all directions. Hence, it does not depend on the width of the water
vessel.

Q Source: Improvisation: Chapter 11: Science 8th NCERT: Page 140

79. Disguised unemployment occurs

a) when the number of unemployed labour exceeds employed labour in


an industry

b) due to the paradox of simultaneous existence of high unemployment


and skilled labour

c) when additional labour do not contribute to the output

d) when there are job vacancies in an industry despite producing full


output

Solution: c)

Explanation: In case of disguised unemployment people appear to be


employed. If a work requiring at most 5 people employs 8 people, then
the extra 3 do not add to the output and are essentially unemployed in
disguise, i.e. under utilization of the extra labour. So, option (c) is
correct.

Option (d) is a case of more efficient utilization of existing labour. So,


wrong.

Option (b) is a general phenomenon seen in developing countries like


India where skill sets do not match with available job demand.


PRELIMS TEST SERIES 2016 MOCK TEST - 1
INSIGHTS

WWW.INSIGHTSONINDIA.COM WWW.INSIGHTSIAS.COM
TEST - 3

Learning: In India, this is more common in agriculture and low


productivity sectors like small households establishments. For e.g. 90%
of industrial units in India face the issue of disguised unemployment.

Factors that contribute to it are: a) large unemployment: b) low skill base


of labour; c) less efficient technology and capital utilization; d) sub-
optimal production management etc.

Q Source: Chapter 2: Economics 9th NCERT: Page 25

80. Which of the following roles/functions are performed by the High


Court in India?

1. Determining the Constitutional validity of any legislation or


action of the executive in the country

2. Giving judgments and directives to protect public interest and


human rights throughout India

Which of the above is/are correct?

a) 1 only

b) 2 only

c) Both 1 and 2

d) None

Solution: c)

Explanation & Learning: The Supreme Court and the High Courts
have the power to interpret the Constitution of the country. They can
declare invalid any law of the legislature or the actions of the executive,
whether at the Union level or at the state level, if they find such a law or
action is against the Constitution. This is known as the judicial review.
So, statement 1 is correct.

The powers and the independence of the Indian judiciary allow it to act
as the guardian of the Fundamental Rights. In recent years the Courts
have given several judgments and directives to protect public interest
and human rights. Anyone can approach the courts if public interest is


PRELIMS TEST SERIES 2016 MOCK TEST - 1
INSIGHTS

WWW.INSIGHTSONINDIA.COM WWW.INSIGHTSIAS.COM
TEST - 3

hurt by the actions of government. This is called public interest


litigation. The courts intervene to prevent the misuse of the
government’s power to make decisions. They check malpractices on the
part of public officials. So, statement 2 is also correct.

Q source: Chapter 5: Democratic Politics

81. The ‘Model Code of Conduct (MCC)’ has been issued by the

a) Supreme Court of India

b) Election Commission of India

c) President of India

d) Parliament

Solution: b)

Explanation & Learning: MCC is not a legally binding document.


Hence, SC cannot issue or enforce it. Option (a) is wrong.

What is it?

It is a set of guidelines laid down by the Election Commission to govern


the conduct of political parties and candidates in the run-up to an
election.

What is the need for such a code of conduct?

• It is intended to provide a level playing field for all political parties,


to keep the campaign fair and healthy, avoid clashes and conflicts
between parties, and ensure peace and order. Its main aim is to
ensure that the ruling party, either at the Centre or in the states,
does not misuse its official position to gain an unfair advantage in
an election.

• The Model Code of Conduct comes into force the moment an


election is announced and remains in force till the results are
declared.


PRELIMS TEST SERIES 2016 MOCK TEST - 1
INSIGHTS

WWW.INSIGHTSONINDIA.COM WWW.INSIGHTSIAS.COM
TEST - 3

• For e.g. one provision of the MCC says, “No party or candidate
shall include in any activity which may aggravate existing
differences or create mutual hatred or cause tension between
different castes and communities, religious or linguistic”

Q source: Improvisation: Chapter 4: Democratic Politics: Page 67

82. Which of the following countries is/are NOT a part of G-20 (Major
economies)?

1. South Africa

2. Morocco

3. Mexico

4. Saudi Arabia

Select the correct answer using the codes below.

a) 3 and 4 only

b) 4 only

c) 2 only

d) 1 only

Solution: c)

Explanation: G20 is an international forum for the governments and


central bank governors from 20 major economies.

The members include 19 individual countries—Argentina, Australia,


Brazil, Canada, China, France, Germany, India, Indonesia, Italy, Japan,
South Korea, Mexico, Russia, Saudi Arabia, South Africa, Turkey, the
United Kingdom and the United States—along with the European Union
(EU).

Learning & Q Source: All questions on regional organizations were


most probably made from here in 2015 Prelims.


PRELIMS TEST SERIES 2016 MOCK TEST - 1
INSIGHTS

WWW.INSIGHTSONINDIA.COM WWW.INSIGHTSIAS.COM
TEST - 3

http://www.mea.gov.in/regional-organisations.htm

Please make sure you have a general idea about all major organizations
mentioned here.

83. To become a law, all bills passed by the Lok Sabha and Rajya Sabha
must be approved by

1. Speaker, Lok Sabha

2. Joint Parliamentary Committee

3. Vice-President

4. President

Select the correct answer using the codes below.

a) 1 and 3 only

b) 2 only

c) 4 only

d) 3 and 4 only

Solution: c)

Explanation: Speaker of Lok Sabha only has to endorse certain money


bills when they are sent to the President. He need not give assent to a bill
to get it passed. Statement 1 is wrong. Same is true for Vice-President,
Chairman of Rajya Sabha, thus statement 3 is also wrong.

A bill is referred to a Joint Parliamentary Committee for examining


controversial provisions in the bill or to give insights on the bill to make
it better. Its recommendations are not binding. Statement 2 is wrong.

Learning: The Parliament consists of two Houses and President of


India. The two Houses are known as the Council of States (Rajya Sabha)
and the House of the People (Lok Sabha).


PRELIMS TEST SERIES 2016 MOCK TEST - 1
INSIGHTS

WWW.INSIGHTSONINDIA.COM WWW.INSIGHTSIAS.COM
TEST - 3

The President of India is a part of the Parliament, although she is not a


member of either House. That is why all laws made in the Houses come
into force only after they receive the assent of the President.

Q source: Chapter 5: Democratic Politics: Page 84

84. Which of the following force(s) is/are both attractive and repulsive?

1. Gravitation Force

2. Electromagnetic force

3. Strong interaction force

Select the correct answer using the codes below.

a) 1 and 2 only

b) 2 and 3 only

c) 1 and 3 only

d) 3 only

Solution: b)

Explanation & Learning:

There are 4 fundamental forces that have been identified. In our present
Universe they have rather different properties.

Properties of the Fundamental Forces

• The strong interaction is very strong, but very short-ranged. It acts


only over ranges of order 10-13 cm and is responsible for holding
the nuclei of atoms together. It is basically attractive, but can be
effectively repulsive in some circumstances.

• The electromagnetic force causes electric and magnetic effects such


as the repulsion between like electrical charges or the interaction
of bar magnets. It is long-ranged, but much weaker than the strong
force. It can be attractive or repulsive, and acts only between pieces
of matter carrying electrical charge.


PRELIMS TEST SERIES 2016 MOCK TEST - 1
INSIGHTS

WWW.INSIGHTSONINDIA.COM WWW.INSIGHTSIAS.COM
TEST - 3

• The weak force is responsible for radioactive decay and neutrino


interactions. It has a very short range and, as its name indicates, it
is very weak.

• The gravitational force is weak, but very long ranged. Furthermore,


it is always attractive, and acts between any two pieces of matter in
the Universe since mass is its source.

Q Source: Improvisation: Chapter 11: Science 8th NCERT: Page 137

85. The Terai region is a

a) plain swampy landscape south of Shiwaliks

b) highland east of Tibet Plateau

c) riverine island supporting high biodiversity

d) hilly region in North-eastern India

Solution: a)

Learning: It was in news due to Nepalese constitution issues.

The Northern plains can be divided into four regions – two are discussed
here. The rivers, after descending from the mountains deposit pebbles in
a narrow belt of about 8 to 16 km in width lying parallel to the slopes of
the Shiwaliks. It is known as bhabar.

All the streams disappear in this bhabar belt. South of this belt, the
streams and rivers re-emerge and create a wet, swampy and marshy
region known as terai. This was a thickly forested region full of wildlife.
The forests have been cleared to create agricultural land and to settle
migrants from Pakistan after partition.

Q Source: Chapter 2: Contemporary India – I: Page 12

86. Extratropical storms are different from Tropical storms in which


way?


PRELIMS TEST SERIES 2016 MOCK TEST - 1
INSIGHTS

WWW.INSIGHTSONINDIA.COM WWW.INSIGHTSIAS.COM
TEST - 3

a) Moisture is carried in lower atmosphere in case of tropical storms,


whereas it is carried in upper atmosphere in extratropical storm

b) tropical storms bring rain, while extratropical storms do not

c) Extratropical storms are caused due to easterly trade winds, whereas


tropical storms are caused without any trade winds

d) None of the above

Solution: a)

Explanation & Learning: Both storms bring rain. So, option (b) is
incorrect.

Extratropical storms are caused due to jet streams, and not due to lower
atmospheric circulations. So, option (c) is incorrect.

About option (d) - This has been explained in another question too.

Jet streams are an important component of upper air circulation which


is dominated by westerly flow. These jet streams blow south of
Himalayas, all through the year except in summer affecting India’s
climate and weather.

The western cyclonic disturbances experienced in the north and north-


western parts of the country are brought in by this westerly flow which
affects rainfall in Northern and north-western India.

Extratropical storms are a global, rather than a localized, phenomena


with moisture usually carried in the upper atmosphere (unlike tropical
storms where it is carried in the lower atmosphere). In the case of the
subcontinent, moisture is sometimes shed as rain when the storm system
encounters the Himalayas.

Q Source: Improvisation: Chapter 4: Contemporary India – I: Page 28

87. China-Pakistan economic corridor (CPEC) project seeks to connect

a) Chah bahar port in Pakistan to the Tibetan region occupied by China

b) Lahore in Pakistan to Beijing in China


PRELIMS TEST SERIES 2016 MOCK TEST - 1
INSIGHTS

WWW.INSIGHTSONINDIA.COM WWW.INSIGHTSIAS.COM
TEST - 3

c) Gwadar port in Pakistan to the Xinjiang region in China

d) Bunker facilities in Islamabad, Pakistan to shipping mouths near East


China Sea

Solution: c)

Explanation: Chah bahar port is being constructed with the support of


India in Iran. Option (a) can be easily eliminated.

Learning:

• CPEC seeks to connect the Gwadar port in Pakistan to the Xinjiang


region in China by means of highways, airways and railways and
also deploy pipeline to transport oil and gas.

• The corridor will also act as a trade route for Chinese goods and
commodities meant for the Middle East and Africa.

• India has already objected to the China Pakistan Economic


Corridor as it passes through the Pakistan occupied Kashmir
(PoK).

• However, China has defended the project saying it is a commercial


venture aimed at improving livelihoods of local people in Pakistan
and strengthening the infrastructure in this region.

Q Source:
http://economictimes.indiatimes.com/news/international/business/chi
na-to-back-pakistan-in-ensuring-security-of-economic-
corridor/articleshow/49767693.cms

88. Antyodaya Anna Yojana (AAY) is a

a) Food for works programme

b) subsidized food entitlements scheme for Below Poverty Line (BPL)


citizens


PRELIMS TEST SERIES 2016 MOCK TEST - 1
INSIGHTS

WWW.INSIGHTSONINDIA.COM WWW.INSIGHTSIAS.COM
TEST - 3

c) food security financial support programme for undernourished


villages in India

d) None of the above

Solution: b)

Explanation & Learning: AAY was launched in December 2000.

Under the scheme one crore of the poorest among the BPL families
covered under the targeted public distribution system were identified.

Poor families were identified by the respective state rural development


departments through a Below Poverty Line (BPL) survey.

Twenty five kilograms of food grains were made available to each eligible
family at a highly subsidised+ rate of Rs 2 per kg for wheat and Rs 3 per
kg for rice.

This quantity has been enhanced from 25 to 35 kgs with effect from April
2002. The scheme has been further expanded twice by additional 50
lakh BPL families in June 2003 and in August 2004. With this increase,
2 crore families have been covered under the AAY.

Q Source: Chapter 4: Economics 9th NCERT: Page 50

89. Which of the following can be a low-cost technique(s) to separate


healthy seeds from damaged seeds?

a) Treating the seeds with starch

b) Floating seeds on water

c) Checking whether the seed has sharp ends

d) All of the above

Solution: b)

Explanation: Starch is a carbohydrate consisting of a large number of


glucose units. It is not useful is checking quality of seeds. Option (a) is
wrong.


PRELIMS TEST SERIES 2016 MOCK TEST - 1
INSIGHTS

WWW.INSIGHTSONINDIA.COM WWW.INSIGHTSIAS.COM
TEST - 3

Damaged seeds become hollow and are thus lighter. Therefore, they float
on water. This is a good method for separating good, healthy seeds from
the damaged ones. Option (b) is correct.

Even healthy seeds of peculiar type may have sharp ends. They cannot be
separated this way. Option (c) is wrong.

Q Source: Chapter 1: Science 8th NCERT: Page 4

90. ‘IMPRINT’ India initiative recently launched by the President of


India relates to

a) Boosting research in the country

b) Imparting skills to young graduates

c) Promoting vernacular literature in urban centres

d) Reviving traditional book presses in financial crisis

Solution: a)

Learning: The initiative is joint venture of Indian Institutes of


Technology (IITs) and Indian Institute of Science (IISC).

IMPRINT India initiative seeks to develop a roadmap for research in


order to solve major engineering and technology challenges in 10
identified technology domains.

These 10 domains are: Health Care, Computer Science and ICT, Advance
Materials, Water Resources and River systems, Sustainable Urban
Design, Defence, Nano-technology Hardware, Environmental Science
and Climate Change and Energy Security.

Q Source: http://pib.nic.in/newsite/mbErel.aspx?relid=130197


PRELIMS TEST SERIES 2016 MOCK TEST - 1
INSIGHTS

WWW.INSIGHTSONINDIA.COM WWW.INSIGHTSIAS.COM
TEST - 3

91. Thal, bhor and the pal ghats passes relate to

a) Vindhya mountain ranges

b) Eastern Ghats

c) Western Ghats

d) Rajmahal Hills

Solution: c)

Learning: The Western Ghats and the Eastern Ghats mark the western
and the eastern edges of the Deccan Plateau respectively.

Western ghats lie parallel to the western coast. They are continuous
(unlike eastern ghats which are discontinuous) and can be crossed
through passes only – like the Thal, Bhor and the Pal Ghats.

The Western Ghats are higher than the Eastern Ghats. Their average
elevation is 900–1600 metres as against 600 metres of the Eastern
Ghats.

Q Source: Chapter 2: Contemporary India – I: Page 13

92. A constitution serves which of following purposes?

1. It is the supreme law which determines the relationship among


citizens living in a territory.

2. It limits the powers of government.

Which of the above is/are correct?

a) 1 only

b) 2 only

c) Both 1 and 2

d) None

Solution: c)


PRELIMS TEST SERIES 2016 MOCK TEST - 1
INSIGHTS

WWW.INSIGHTSONINDIA.COM WWW.INSIGHTSIAS.COM
TEST - 3

Explanation & Learning: The constitution of a country is a set of


written rules that are accepted by all people living together in a country.
Constitution is the supreme law that determines the relationship among
people living in a territory (called citizens) and also the relationship
between the people and government. A constitution does many things:

• First, it generates a degree of trust and coordination that is


necessary for different kind of people to live together;

• Second, it specifies how the government will be constituted, who


will have power to take which decisions;

• Third, it lays down limits on the powers of the government and


tells us what the rights of the citizens are; and

• Fourth, it expresses the aspirations of the people about creating a


good society.

Q source: Chapter 2: Democratic Politics: Based on concepts

93. Consider the following about the Asian Infrastructure Investment


bank (AIIB).

1. It admits only Asian member countries.

2. It is led by China.

3. All members have equal voting rights in AIIB.

Select the correct answer using the codes below.

a) 1 and 2 only

b) 2 only

c) 2 and 3 only

d) 1 and 3 only

Solution: b)


PRELIMS TEST SERIES 2016 MOCK TEST - 1
INSIGHTS

WWW.INSIGHTSONINDIA.COM WWW.INSIGHTSIAS.COM
TEST - 3

Explanation: Even non-regional (Asian countries) can be its members.


However, their vote share will be lesser than Asian members. So,
statements 1 and 3 are incorrect.

Learning: It is a multilateral development bank (MDB)

Asia has a massive infrastructure funding gap around $8 trillion. ADB


($160 billion) and World Bank ($223 billion) even together cannot fill
the funding gap. Hence, the need for AIIB.

Both the ADB (formed in 1966) and AIIB (formed in 2015) are responses
by Asia’s leading economic power. Motivation lies in improving global
economic governance structures e.g. IMF, World Bank etc.

Q Source:
http://zeenews.india.com/business/news/international/china-ratifies-
aiib-agreement_1818247.html

94. Mangroves do NOT occur in which state(s) in India?

1. Andhra Pradesh

2. Goa

3. Odisha

4. Meghalaya

Select the correct answer using the codes below.

a) 2 only

b) 1 and 3 only

c) 2 and 4 only

d) 4 only

Solution: d)

Explanation & Learning: The mangrove tidal forests are found in the
areas of coasts influenced by tides. Dense mangroves are the common
varieties with roots of the plants submerged under water. The deltas of


PRELIMS TEST SERIES 2016 MOCK TEST - 1
INSIGHTS

WWW.INSIGHTSONINDIA.COM WWW.INSIGHTSIAS.COM
TEST - 3

the Ganga, the Mahanadi, the Krishna, the Godavari and the Kaveri are
covered by such vegetation.

Experiences have proved that the presence of mangrove ecosystems on


coastline save lives and property during natural hazards such as
cyclones, storm surges and erosion. These ecosystems are also well
known for their economic importance.

They are breeding, feeding and nursery grounds for many estuarine and
marine organisms. Hence, these areas are used for captive and culture
fisheries

Refer to the link to know state, UT wise cover and other information
about Mangroves.

http://agritech.tnau.ac.in/forestry/forest_mangrove_index.html#mang
roves

Q Source: Chapter 5: Contemporary India – I: Page 47

95. Which of the following phenomenon can be described with the help
of electromagnetism?

1. Friction

2. Rainbows

3. Lightning

Select the correct answer using the codes below.

a) 1 only

b) 2 and 3 only

c) 3 only

d) All of the above

Solution: d)


PRELIMS TEST SERIES 2016 MOCK TEST - 1
INSIGHTS

WWW.INSIGHTSONINDIA.COM WWW.INSIGHTSIAS.COM
TEST - 3

Explanation & Learning: All three are caused due to


electromagnetism, a fundamental force in nature.

Friction:

• Electromagnetic force is the force which holds the atoms together


and keeps the electrons from flying off somewhere away from the
atoms nucleus and also holds the atoms together to form
molecules. Certain molecules are going to attract to each other
increasing frictional forces and some molecules repel reducing
frictional forces.

• For e.g. Teflon or PTFE is a very slippery material (having less


friction). It is a long string of carbon atoms joined together with
two fluorine atoms attached to each carbon atom. Fluorine, when
attached to a molecule doesn't like any other molecule around it. It
repels any other molecule even other molecules with fluorine
atoms, hence its low coefficient of friction or slipperiness.

• Hence, statement 1 is correct.

Rainbows:

Rainbow is caused by reflection, refraction and dispersion of light in


water droplets resulting in a spectrum of light (electromagnetic)
appearing in the sky. So, statement 2 is also correct.

Lightning: It is an electrostatic discharge that travels between two


charged regions. So, statement 3 is correct.

Q Source: Improvisation: Chapter 11: Science 8th NCERT: Page 137

96. Consider the following with reference to ‘Over-the-top services’ in


India.

1. It is provided directly by the Internet Service Providers (ISPs).


PRELIMS TEST SERIES 2016 MOCK TEST - 1
INSIGHTS

WWW.INSIGHTSONINDIA.COM WWW.INSIGHTSIAS.COM
TEST - 3

2. They are presently regulated by the Ministry of Information &


Broadcasting.

Which of the above is/are correct?

a) 1 only

b) 2 only

c) Both 1 and 2

d) None

Solution: d)

Explanation: OTT refers to a service that is provided on top of the


internet service. For e.g. you may use the Whatsapp calling feather on a
internet network provided by Airtel. But, Airtel as an ISP, is NOT
concerned presently with how you use its internet. So, statement 1 is
wrong.

There are proposals to regulate OTTs after the telecom companies


lobbied. TRAI also released a consultation paper in March 2015 on OTT
regulatory framework. But, no consensus has come on the regulation. So,
statement 2 is also wrong.

Moreover, you should understand that the Ministry of I&B will not
regulate telecom services. It is in the domain of the Telecom Regulatory
Authority of India (TRAI).

The controversy over OTTs is because companies like Whatsapp and


Skype use the networks of the ISPs to expand their business, even while
the ISPs suffer loss of revenue due to shift of consumers from telecom
services to OTTs like Whatsapp messaging or calling.

Learning: The Internet Service provider (ISP) may be aware of the


contents of the Internet Protocol packets but is not responsible for, nor
able to control, the viewing abilities, copyrights, and/or other
redistribution of the content.

Consumers can access OTT content through any internet-connected


devices such as desktop and laptop computers, gaming consoles (such as
the PlayStation), set-top boxes etc.


PRELIMS TEST SERIES 2016 MOCK TEST - 1
INSIGHTS

WWW.INSIGHTSONINDIA.COM WWW.INSIGHTSIAS.COM
TEST - 3

Q Source: Yojana Sept 2015 – DO YOU KNOW?

97. Deen Dayal Upadhyay Grameen Jyoti Yojana (DDUGJY) does NOT
focus on

a) Providing electricity to all villages in India

b) Separating feeder lines for rural industrial and other uses

c) Reduction of transmission and distribution losses

d) Bridging revenue loss by effective monitoring of electricity usage

Solution: b)

Explanation: DDUGJY does NOT separate feeder lines for industrial


and other uses. It separates the power of rural farmer households from
Main supply to ensure efficient power supply to farmers. So, option (b) is
wrong.

Option (d) is correct as DDUGJY intends to cover revenue losses through


metering.

Learning: The new scheme of Deen Dayal Upadhyaya Gram Jyoti


Yojana (DDUGJY), recently approved by the Union Government draws
its inspiration from the similar pioneering scheme implemented by the
Government of Gujarat.

• This scheme will enable to initiate much awaited reforms in the


rural areas.
• It focuses on feeder separation (rural households & agricultural)
and strengthening of sub-transmission & distribution
infrastructure including metering at all levels in rural areas.
• This will help in providing round the clock power to rural
households and adequate power to agricultural consumers .


PRELIMS TEST SERIES 2016 MOCK TEST - 1
INSIGHTS

WWW.INSIGHTSONINDIA.COM WWW.INSIGHTSIAS.COM
TEST - 3

• The earlier scheme for rural electrification viz. Rajiv Gandhi


Grameen Vidyutikaran Yojana (RGGVY) has been subsumed in the
new scheme as its rural electrification component

Q Source: Yojana Sept 2015 – DEVELOPMENT ROADMAP (last page)

98. The implementation of the ‘Smart Cities Mission’ at the City level will
be done by a Special Purpose Vehicle (SPV). Which of the following
statements is INCORRECT about SPVs?

a) It will be a limited company incorporated under the Companies Act.

b) SPVs will not include any private sector participation in ownership.

c) Government of India will provide funds to the SPVs to implement the


project.

d) Both the State government and Urban Local Body will control the
SPVs.

Solution: b)

Explanation: The private sector or financial institutions could be


considered for taking equity stake in the SPV, provided the shareholding
pattern of 50:50 of the State/UT and the ULB is maintained and the
State/UT and the ULB together have majority shareholding and control
of the SPV. So, option (b) is incorrect, and (d) is correct.

Learning: The SPV will plan, appraise, approve, release funds,


implement, manage, operate, monitor and evaluate the Smart City
development projects.

• Each smart city will have a SPV which will be headed by a full time
CEO and have nominees of Central Government, State
Government and ULB on its Board.
• The States/ULBs shall ensure that,


PRELIMS TEST SERIES 2016 MOCK TEST - 1
INSIGHTS

WWW.INSIGHTSONINDIA.COM WWW.INSIGHTSIAS.COM
TEST - 3

a) a dedicated and substantial revenue stream is made available


to the SPV so as to make it self-sustainable and could evolve
its own credit worthiness for raising additional resources
from the market and
b) Government contribution for Smart City is used only to
create infrastructure that has public benefit outcomes.
• The execution of projects may be done through joint ventures,
subsidiaries, public-private partnership (PPP), turnkey contracts,
etc suitably dovetailed with revenue streams.

Q Source: Improvisation: Yojana Sept 2015 – DEVELOPMENT


ROADMAP (last page)

http://smartcities.gov.in/writereaddata/SPVs.pdf

99. Which of the following statements is INCORRECT about ‘Home


Guards’?

a) It is a voluntary force.

b) It assists the police forces in maintaining internal security.

c) It serves as an auxiliary to the Border Security Force (BSF).

d) It does not engage itself in socio-economic and welfare activities.

Solution: d)

Explanation: It was first raised in India 1946 to assist the police in


controlling civil disturbance and communal riots; states followed suit
and raised their own Voluntary organizations. After the Chinese
Aggression (1962), all states merged their Home guards into one single
unit. So, option (a) and (b) are correct.


PRELIMS TEST SERIES 2016 MOCK TEST - 1
INSIGHTS

WWW.INSIGHTSONINDIA.COM WWW.INSIGHTSIAS.COM
TEST - 3

Home Guards are of two types - rural and urban. In border States,
Border Wing Home Guards Bns. have also been raised, which serve as an
auxiliary to the Border Security Force. So, option (c) is correct.

It not only helps in maintenance of internal security and in disaster


management, but also assists the administration in protecting weaker
sections, participate in socio-economic and welfare activities and
perform Civil Defence duties. So, option (d) is incorrect.

Learning & Q Source: http://ndrfandcd.gov.in/CMS/AboutUs.aspx

The question was improvised from Yojana – NORTH EAST DIARY


(NDRF).

100. Consider the following statements.

1. It is located on the edge of the Eastern Himalaya biodiversity


hotspot.

2. It is a World Heritage Site

3. Vegetation consists of semi-evergreen forests, tropical and


subtropical moist broadleaf forests, Savannas and shrublands.

4. The park area is circumscribed by the Brahmaputra River.

The above refer to which National park in India?

a) Nokrek National Park

b) Keibul Lamjao National Park

c) Namdapha National Park

d) Kaziranga National Park

Solution: d)


PRELIMS TEST SERIES 2016 MOCK TEST - 1
INSIGHTS

WWW.INSIGHTSONINDIA.COM WWW.INSIGHTSIAS.COM
TEST - 3

Explanation: From statement 4, you can understand that the park is


in North-eastern India. But, all parks mentioned in options are from
North-east.

But, Brahmaputra does not pass through Nokrek and Keibul Lamjao
national park. Later is situated in the Loktak lake. So, options (b) and (a)
can be eliminated.

The answer then must be either (c) or (d). However, Nokrek is not a
world heritage site, but comes under UNESCO Bipsphere Reserves. So,
answer is (d), Kaziranga.


PRELIMS TEST SERIES 2016 MOCK TEST - 1
INSIGHTS


INSIGHTS ON INDIA MOCK PRELIMINARY EXAM - 2016


INSIGHTS ON INDIA MOCK TEST – 03

GENERAL STUDIES
PAPER-I
Time Allowed: 2 Hours Maximum Marks: 200

INSTRUCTIONS
1. IMMEDITELY AFTER THE COMMENCEMENT OF THE EXAMINATION, YOU SHOULD
CHECK THAT THIS TEST BOOKLET DOES NOT HAVE ANY UNPRINTED OR TORN OR MISSING
PAGES OR ITEMS, ETC. IF SO, GET IT REPLACED BY A COMPLETE TEST BOOKLET.
2. You have to enter your Roll Number on the Test
Booklet in the Box provided alongside. DO NOT
Write anything else on the Test Booklet.
4. This Test Booklet contains 100 items (questions). Each item is printed only in English. Each item
comprises four responses (answers). You will select the response which you want to mark on the
Answer Sheet. In case you feel that there is more than one correct response, mark the response which
you consider the best. In any case, choose ONLY ONE response for each item.
5. You have to mark all your responses ONLY on the separate Answer Sheet provided. See directions in
the Answer Sheet.
6. All items carry equal marks.
7. Before you proceed to mark in the Answer Sheet the response to various items in the Test Booklet, you
have to fill in some particulars in the Answer Sheet as per instructions sent to you with your Admission
Certificate.
8. After you have completed filling in all your responses on the Answer Sheet and the examination has
concluded, you should hand over to the Invigilator only the Answer Sheet. You are permitted to take
away with you the Test Booklet.
9. Sheets for rough work are appended in the Test Booklet at the end.
10. Penalty for wrong answers :
THERE WILL BE PENALTY FOR WRONG ANSWERS MARKED BY A CANDIDATE IN THE
OBJECTIVE TYPE QUESTION PAPERS.
(i) There are four alternatives for the answer to every question. For each question for which a
wrong answer has been given by the candidate, one-third of the marks assigned to that
question will be deducted as penalty.
(ii) If a candidate gives more than one answer, it will be treated as a wrong answer even if one of
the given answers happens to be correct and there will be same penalty as above to that
question.
(iii) If a question is left blank, i.e., no answer is given by the candidate, there will be no penalty for
that question.

http://www.insightsonindia.com

INSIGHTS ON INDIA MOCK TEST SERIES FOR CIVIL SERVICES PRELIMINARY EXAM
2016

http://www.insightsonindia.com INSIGHTS Page 1





4. Consider the following
1. The difference between a statements.
Sanctuary and a National Park
mainly lies in Assertion (A): Coastal regions in
a) Area, size and government India do not have irrigation facilities.
control Reason (R): Farming in coastal
b) PRIs v/s state government areas is completely rain-fed due to high
control rainfall near the sea.
c) Rights of people living inside
d) Regulation of poaching and In the context of the above, which of
hunting these is correct?

a) A is correct, and R is an
appropriate explanation of A.
2. Wular Lake, the largest b) A is correct, but R is not an
freshwater lake in India, has appropriate explanation of A.
been formed due to c) A is correct, but R is incorrect.
a) Flooding of many small d) Both A and R are incorrect.
riverine islands
b) Ox-bow lakes
c) Filling of glacier basin
5. Consider the following
d) Tectonic activity
statements about Food and
Agriculture Organization (FAO).
1. It is an agency of the United
3. Which of the following can be Nations
called as ‘factors of production’? 2. It serves only developing and
1. Human labour Least Developed Countries
2. Land (LDCs).
3. Consumer goods 3. It created the Codex
4. Capital goods Alimentarius Commission to
Select the correct answer using the develop food standards.
codes below. Select the correct answer using the
a) 1, 2 and 3 only codes below.
b) 2, 3 and 4 only a) 2 and 3 only
c) 1 only b) 1 only
d) 1, 2 and 4 only c) 1 and 3 only
d) 2 only

http://www.insightsonindia.com INSIGHTS Page 2





a) 1, 2 and 6 only
6. Use of chemical fertilizers is b) 3, 4 and 5 only
discouraged because c) 2, 3, 4, 5 and 6 only
1. Its minerals take long time to d) All of the above
become available to plants.
2. It may kill soil micro-
organisms. 8. Who is the ‘Visitor’ of Central
3. Agricultural run-off may cause Universities in India?
eutrophication of fresh water a) Prime Minister of India
bodies b) President of India
4. High levels of fertilizer may c) An Academician appointed by
cause the breakdown of the Union Ministry of Human
symbiotic relationships Resource Development
between plant roots and (MHRD)
mycorrhizal fungi. d) Chairman, University Grants
Commission (UGC)
Select the correct answer using the
codes below.

a) 2 only 9. Which of the following are MDG


b) 2 and 3 only 2015 targets which have been
c) 1, 3 and 4 only achieved by India?
d) 2, 3 and 4 only 1. Universal primary education
2. Reverse the spread of
HIV/AIDS
7. Apart from Article 21A, which of 3. Eliminate poverty from urban
the following constitutional areas
articles have a bearing on
education? Select the correct answer using the
codes below.
1. Article 3
2. Article 13 a) 1 only
3. Article 28 b) 1 and 3 only
4. Article 30 c) 2 only
5. Article 51A d) 2 and 3 only
6. Article 243G

Select the correct answer using the


codes below.

http://www.insightsonindia.com INSIGHTS Page 3





10. How has India benefitted following statements about it is
from signing the Indus water INCORRECT?
treaty with Pakistan? a) Any type of skilled or
a) The treaty declares India to be unskilled work can be allotted
the owner of Indus water under MGNREGA.
resources. b) One of the aims of MGNREGA
b) The treaty bans Pakistan from is to create durable assets.
constructing and c) MGNREGA is to be
embankments or dams on the implemented mainly by gram
Indus river system. panchayats (GPs) as the
c) As per the treaty, to obtain involvement of contractors is
water from Indus, Pakistan is banned.
obliged to share it irrigation d) If work is not provided within
infrastructure with India. 15 days of applying, applicants
d) None of the above are entitled to an
unemployment allowance.

11. Which of the following


organizations publish the 13. Apart from Supreme Court, writs
‘International Poverty Line’? to enforce fundamental rights
a) United Nations Development can be issued by
Program (UNDP) 1. District Courts
b) World Food Programme 2. National Police Commission
(WFP) 3. Lok Adalats
c) International Labour
Select the correct answer using the
Organization (ILO)
codes below.
d) World Bank
a) 1, 2 and 3 only
b) 1 and 3 only
12.The Mahatma Gandhi National c) 2 only
Rural Employment Guarantee d) None of the above
Act (MGNREGA) intends to
enhance livelihood security in
rural areas by providing at least
100 days of guaranteed wage
employment in a financial year to
willing households. Which of the

http://www.insightsonindia.com INSIGHTS Page 4





14.Village Grain Banks Scheme is a) 1 and 2 only
concerned with b) 2 and 3 only
a) Providing loans to small and c) 1, 3 and 4 only
marginal farmers in case of d) All of the above
crop loss
b) Providing facility to borrow
food grains in food scarce and 17. Gold Monetisation Scheme
calamity hit areas (GMS) 2015 is available to
c) Giving loans for food supply a) Only Non-Resident Indians
chain creation in rural areas (NRIs)
d) Providing a single window b) Both NRIs and resident
mechanism for farmers to Indians
procure agricultural inputs c) Only Indian citizens with
income above a certain
threshold
15. Wangala Festival is d) All resident Indians
a) a major religious procession of
Khonds in Sikkim
b) annual post-harvest festival of 18. Consider the following
the Garos tribes of North east about Rhizobium.
India 1. It colonises legume plant’s
c) a literary event root nodules.
commemorating the writings 2. It provides food to those
of vernacular language symbiotic species which
authors of eastern India cannot manufacture
d) None of the above food.
3. It grows by the technique of
vegetative propagation.

16.In India, Minimum Support Which of the above is/are correct?


Prices (MSP) are announced for a) 1 only
1. Pulses b) 2 and 3 only
2. Oilseeds c) 1 and 3 only
3. Tea d) 2 only
4. Coffee

Select the correct answer using the


codes below.

http://www.insightsonindia.com INSIGHTS Page 5





19.Which authority in India 22. International Covenant on
maintains the Voter’s list for a Economic, Social and Cultural
particular state? Rights does NOT recognize which
a) Chief Electoral Officer of the human right?
State a) Right to work opportunity
b) District Magistrates of the b) Right to social security and
concerned areas insurance
c) Chief Secretary c) Right to adequate housing
d) Zila Parishad d) Right against indulgement in
war by the State
20. Which of these may NOT
be a good reason to say that 23. Why is the colour of a
elections in a certain country are candle flame yellow, whereas
democratic? flame of a kitchen stove is blue?
a) If it has very large voter base. a) Temperature in candle flame
b) If an independent and is higher than that of kitchen
powerful body supervises stove
elections. b) The amount of fuel burning
c) If all candidates have a fair per unit of time is very less in
chance of competing and a candle as compared to a
winning in elections. kitchen stove
d) If the losing parties peacefully c) Fuel burns completely in a
accept the electoral verdict. stove and has better exposure
to oxygen than in a candle
21.Magnesium deficiency in plants d) All of the above
can cause degradation of
chlorophyll in old leaves. In what 24. A ‘by-election’ is held when
type of soil is Magnesium a) A party candidate during
deficiency most likely to occur elections withdraws or dies
due to leaching? b) Vacancy caused by death or
a) Red soil with high loam resignation of a legislator is to
content be filled between general
b) Strongly acidic, light and elections
sandy soils c) The Election Commission of
c) Clayey soil which has high India thinks it to be in the
mineral adhesion best-interest of the nation.
d) Any soil in areas of moderate d) There is no decisive winner in
rainfall a general election
http://www.insightsonindia.com INSIGHTS Page 6



27. Consider the following
statements.
25. In a wildlife sanctuary,
1. It lies in the Satpura Range of
generally which of the following
Madhya Pradesh.
activities are regulated?
2. It includes Bori sanctuary and
1. Plantation and cultivation
Satpura national park.
2. Grazing
3. It is a part of the World
3. Tourism
Network of Biosphere
4. Scientific research
Reserves, UNESCO.
Select the correct answer using the
The above statements refer to?
codes below.
a) Panna Biosphere Reserve
a) 1 and 4 only
b) Panchmari Biosphere Reserve
b) 2 and 3 only
c) Achanakmar-Amarkantak
c) 1 and 2 only
Biosphere Reserve
d) All of the above
d) Simlipal Biosphere Reserve

26. Which of the following


28. Why the difference
describes the ‘Mission’ of the
between the durations of day and
Atomic Energy Regulatory Board
night is felt hardly at
most appropriately?
Kanyakumari but not so much in
a) Safeguarding public health
Kashmir?
and environment from
a) Due to difference of altitude of
hazards of nuclear energy
two places above Mean Sea
b) Addressing energy security of
Level (MSL)
the nations by establishing
b) Due to difference in latitudes
high end nuclear power
of two places
stations
c) Due to difference in longitudes
c) Supervising quality research
of two places
in matters of fundamental
d) Due to difference in proximity
nuclear energy to serve
with ocean of the two places
national interest
d) Oversee the technical
operations of nuclear power
centres in India

http://www.insightsonindia.com INSIGHTS Page 7





29. Consider the following 2. Electromagnetic force is a
statements. short-range force.
1. Lok Sabha exercises more
Which of the above is/are correct?
powers in matters of money
bills than Rajya Sabha. a) 1 only
2. Council of Ministers can be b) 2 only
appointed only by the Lok c) Both 1 and 2
Sabha. d) None
Which of the above is/are correct?

a) 1 only 32. APJ Abdul Kalam Amrut


b) 2 only Yojna relates to
c) Both 1 and 2 a) Providing one cooked meal to
d) None pregnant, lactating women in
tribal areas
b) Improving the vaccination
30. Consider the following rates in rural India to tackle
statements about In-vitro preventable diseases
fertilization. c) Registering all Mid-Day meal
1. It leads to irreversible genetic serving NGOs and providing
changes in the embryo. them single window
2. It is illegal in India. administrative clearances
d) None of the above
Which of the above is/are correct?

a) 1 only
b) 2 only 33. The National River
c) Both 1 and 2 Conservation Directorate
d) None (NRCD) is under
a) Ministry of Environment and
Forests
31. Even though electromagnetism is b) Ministry of Drinking Water
far stronger than gravitation, and Sanitation
electrostatic attraction is not c) Ministry of Water Resources
relevant for large celestial bodies. d) Ministry of Urban
Why? Development
1. Net electric charge of such
bodies is zero.

http://www.insightsonindia.com INSIGHTS Page 8





34. As compared to their 36. Eyes can be donated by
Himalayan counterparts, people
Peninsular Rivers have 1. Of any age
1. Shorter courses 2. Using spectacles
2. Deeper beds 3. Suffering from AIDS
3. Higher gradient load 4. Suffering from Tetanus and
Cholera
Select the correct answer using the
codes below. Select the correct answer using the
codes below.
a) 1 only
b) 2 and 3 only a) 1 and 3 only
c) 1 and 3 only b) 1, 2 and 4 only
d) All of the above c) 2 and 4 only
d) 1 and 2 only

35. Which of the following


institutions can expand and 37. If there is a difference
increase the scope of between the two Houses of
fundamental rights in the Parliament on ordinary
constitution? legislation, the matter
1. President of India a) must be referred to the
2. Supreme Court President of India, whose
3. High Court opinion shall be final and
4. Parliament binding
5. National Human Rights b) can be referred to the
Commission Supreme Court of India for its
advisory jurisdiction
Select the correct answer using the
c) will be referred to a joint
codes below.
sitting of the Parliament
a) 2 and 4 only convened by the President
b) 2, 3 and 4 only d) will be settled through a Joint
c) 1, 2, 4 and 5 only Parliamentary Committee
d) 1 and 4 only setup by both the houses

http://www.insightsonindia.com INSIGHTS Page 9





38. When the earth crosses the 2. Upper air circulation
tail of a comet, it indicates 3. Tropical easterly jet stream
a) an impending solar spot cycle
Select the correct answer using the
b) possible sight of a meteor
codes below.
shower
c) an impending high tide event a) 1 only
in oceans b) 2 and 3 only
d) None of the above c) 3 only
d) All of the above
39. Which of the following
institutions resolves the disputes 42. Which of the following is
between Centre and States NOT essential to have and
arising out of official decisions? sustain a democracy?
a) Union Finance Commission a) Protection of the fundamental
(UFC) rights of all citizens
b) Supreme Court (SC) b) Freedom to express views,
c) Central Administrative freedom to organise and
Tribunal (CAT) freedom to protest
d) Chairman, Rajya Sabha c) Political equality of citizens
d) A written constitution
40. Water can be considered
polluted due to 43. Sandalwood, khair, kusum,
1. High concentration of arjun and mulberry which are
nutrients commercially important tree
2. Higher temperature species are found in
3. Presence of larvae eating fish 1. Tropical deciduous forests
2. Tropical evergreen forests
Select the correct answer using the
3. Montane forests
codes below.
Select the correct answer using the
a) 1 and 2 only
codes below.
b) 2 and 3 only
c) 1 and 3 only a) 1 and 2 only
d) None of the above b) 1 only
c) 2 and 3 only
41.The climate and associated d) 2 only
weather conditions in India are
governed by
1. Pressure and surface winds
http://www.insightsonindia.com INSIGHTS Page 10



44. Which of the following
organizations provide direct
47. Large scale deforestation in
loans to governments?
a forest can cause fertility of soil
a) International Monetary Fund
to decline. Why?
(IMF)
1. Soil can be leached easily due
b) World Bank (WB)
to heavy rain
c) Asian Development Bank
2. Microbial activity in soil
(ADB)
increases significantly
d) All of the above
3. Nutrient content of the soil
changes.
45. In 2005, the Supreme
Court has declared the Select the correct answer using the
Presidential Proclamation codes below.
dissolving the Bihar Assembly as
unconstitutional. Among the a) 1 and 2 only
following, which democratic b) 2 and 3 only
feature of Indian polity does this c) 1 and 3 only
case safeguards? d) 1 only
a) Respect for Rights
b) Rule of Law
c) Right to Public office 48. Consider the following
d) Universal Adult Franchise about Antibiotics.
1. They are used against both
46. National Green Tribunal bacteria and viruses.
(NGT) has recently banned straw 2. When taken unnecessarily
burning in some North-western they may kill the beneficial
states of India because it emitted bacteria in the body.
1. Lead 3. Regular use of high doses of
2. Arsenic antibiotics may make them
3. Particulate Matter ineffective.
4. Hexafluroine Select the correct answer using the
Select the correct answer using the codes below.
codes below. a) 2 only
a) 1 and 2 only b) 1 and 3 only
b) 3 only c) 1 and 2 only
c) 3 and 4 only d) 2 and 3 only
d) All of the above
http://www.insightsonindia.com INSIGHTS Page 11



49. Main tributaries of Kaveri 51. Why are the western slopes of the
are Western Ghats covered with
1. Bhavani thick forests and not the eastern
2. Amravati slopes?
3. Brahmani 1. Eastern side falls in rain-
4. Baitarini shadow zone
2. No protected forests exist on
Select the correct answer using the
the eastern slopes.
codes below.
3. Soil in eastern slope cannot
a) 1 and 2 only support vegetation due to
b) 2 and 3 only absence of nutrients.
c) 3 and 4 only
Select the correct answer using the
d) 1 and 4 only
codes below.

50. Consider the following a) 1 and 2 only


about the Election Commission b) 2 and 3 only
of India (ECI). c) 1 only
1. The Chief Election d) 1 and 3 only
Commissioner (CEC) is
appointed by the President of 52. Consider the following
India. about blue-green algae.
2. It enjoys the same kind of 1. They can fix atmospheric
independence that the nitrogen in aerobic conditions.
judiciary enjoys. 2. They can be used as bio-
3. Once appointed, the Chief fertilizer.
Election Commissioner is fully 3. They do not perform
answerable to the photosynthesis unlike normal
government for its powers and algae.
responsibilities. 4. They can be used to produce
algal biofuel.
Select the correct answer using the
codes below. Select the correct answer using the
codes below.
a) 1 only
b) 2 and 3 only a) 1, 2 and 4 only
c) 1 and 3 only b) 2 and 4 only
d) 1 and 2 only c) 2 and 3 only
d) All of the above

http://www.insightsonindia.com INSIGHTS Page 12





53. The highest formal a) 1 and 3 only
authority in India is carried by b) 2 and 4 only
a) Prime Minister of India c) 1, 2 and 3 only
b) President of India d) 1, 3 and 4 only
c) Cabinet Secretary
d) Heads of independent 57. The last natural refuge of
Constitutional bodies the endangered Manipur brow-
antlered deer (Sangai) is in
54. Which of the following a) Loktak Lake
planets in the solar system are b) Hajong Lake
less dense than water? c) Sendra Lake
a) Mercury, Venus and Saturn d) Kolleru Lake
b) Venus and Neptune
c) Mars and Jupiter 58. Which of the following
d) Saturn only statements about the provisions
of the Forest Conservation Act
(1980) is CORRECT?
55. The Council of Ministers is a) State governments cannot de-
held accountable to and reserve forests without the
controlled by prior approval of Central
a) Both houses of Parliament Government.
b) Chairman, Rajya Sabha b) It consolidates the law relating
c) Lok Sabha to forests, the transit of forest-
d) President of India produce.
c) It recognizes the rights of
traditional forest dwellers over
56. Carbon monoxide can be the forest areas.
produced from or found in d) National parks and biosphere
1. Incomplete burning of petrol reserves are constituted under
and diesel this Act.
2. Naturally in groundwater and
ice caps
3. Operation of a kitchen stove in 59. Which of the following
enclosed space rivers do NOT make ‘delta’?
4. Eruption of volcanoes 1. Mahanadi
2. Tapi
Select the correct answer using the
3. Narmada
codes below.
4. Godavari
http://www.insightsonindia.com INSIGHTS Page 13



Select the correct answer using the 1. The Commission is
codes below. independent of the
government.
a) 1 and 2 only 2. It is a constitutional authority.
b) 2 and 3 only 3. Its recommendations are
c) 3 and 4 only binding on the courts but not
d) 1 and 4 only government.
4. There is no fee to approach
the NHRC.
58. Right to Constitutional Remedies is Select the correct answer using the
codes below.
a) A fundamental right
b) A constitutional right a) 1 only
c) A Legal right b) 1 and 4 only
d) A right based on the directions c) 1, 3 and 4 only
issued by the executive d) 2 and 3 only

62. The shape of a motor


60. Consider the following vehicle affects its
about the Indian Agricultural 1. Fuel efficiency
Research Institute (IARI). 2. Speed
1. It has been established under 3. Overall performance
an Act of the Parliament. Select the correct answer using the
2. It did not contribute to the codes below.
Indian "Green Revolution" of
a) 1 and 2 only
the 1970s as it was established
b) 2 and 3 only
after it. c) 2 only
d) All of the above
Which of the above is/are correct?

a) 1 only
b) 2 only 63. Which of the following
c) Both 1 and 2 problem(s) occur when using dry
d) None ice as a fire extinguisher?
1. It causes harm electrical
equipment nearby resulting in
61.Consider the following about fire.
National Human Rights 2. It aggravates fire situations
Commission. caused due to oil spills.

Which of the above is/are correct?

http://www.insightsonindia.com INSIGHTS Page 14





a) 1 only c) An area of relatively low
b) 2 only atmospheric pressure, which
c) Both 1 and 2 is found mainly in temperate
regions
d) None
d) A narrow, shallow, elongated
basin with a sinking bottom
64. Red Data Book, IUCN, where rivers deposited
keeps a record of all the sediments
endangered animals and plants.
Which of the following 66. Decennial Census exercises
institutions contribute to the red in India are conducted and
list? administered under?
1. BirdLife International a) Ministry of Planning
2. World Conservation b) Ministry of Statistics and
Monitoring Centre Programme Implementation
3. Greenpeace c) Ministry of Home Affairs
4. Environmental Defense Fund d) Ministry of Social Justice and
Select the correct answer using the Empowerment
codes below.
67. Exotic plants of an area
a) 3 and 4 only refer to
b) 1 and 2 only a) Endemic or indigenous
c) 1, 3 and 4 only vegetation of that area
d) 2, 3 and 4 only b) Vegetation that is not native
and has come from outside
65. Suez Canal allows ships to c) Naturally growing vegetation
travel between which has been left
a) North and South America undisturbed by humans for a
b) Europe and Russia long time
c) Europe and South Asia d) Vegetation that grows in a
d) East Africa to South-East Asia specific season after a long
time

65. What describes ‘geosynclines’?


68. When on election duty,
a) A linear break in rocks of the government officers work under
earth’s crust along which the control of
frequent displacement occurs a) The concerned District
b) A mass of sand and soil that
Administration
moves slowly under the
b) State Planning Board
influence of gravity along a
c) Chief Secretary of the State
confined course
d) Election Commission

http://www.insightsonindia.com INSIGHTS Page 15





69. Which of these states have 72. How does ‘relief’ play a role
more than 50% of land under in determining the climate of a
forest cover? place?
1. By blocking or allowing wind
1. Himachal Pradesh
streams
2. Odisha 2. By increasing continentality
3. Sikkim
4. Karnataka Which of the above is/are correct?

Select the correct answer using the a) 1 only


b) 2 only
codes below. c) Both 1 and 2
a) 2, 3 and 4 only d) None
b) 1, 2 and 3 only
73. Which of the following is
c) 1 and 4 only NOT a greenhouse gas?
d) 1 and 3 only a) Nitrogen
b) Ozone
70. Which United Nations c) Methane
Organ is charged with the d) Water vapour
maintenance of international
peace and security as well as 74. Westerly cyclonic
disturbances affect the climate in
recommending admission of new
a) Southern India
members to the United Nations b) North and north-western
(UN)? India
a) UN Secretariat c) Eastern India
b) UN Trusteeship Council d) South-eastern India
c) International Court of Justice
d) UN Security Council
75. ‘UDAY’ scheme launched
recently by the Union Cabinet
71. The term ‘Green Tax’ refers to relates to
1. Excise taxes on environmental a) Consolidation and revamping
pollutants of Prime Minister’s Fifteen
2. Taxes on goods whose use Point Programme
produces such pollutants b) Financial restructuring of debt
Which of the above is/are correct? of power distribution
companies
a) 1 only c) Generating employment in
b) 2 only left-wing extremism affected
c) Both 1 and 2 areas
d) None d) Making all government offices
energy efficient

http://www.insightsonindia.com INSIGHTS Page 16





b) 2 and 3 only
76. Inter Tropical Convergence c) 1 only
Zone is d) 2 only
a) A zone from where all tropical
cyclones originate 79. Disguised unemployment
b) A Zone from where trade occurs
winds emerge and travel a) when the number of
towards poles unemployed labour exceeds
c) A broad trough of low employed labour in an
pressure in equatorial industry
latitudes b) due to the paradox of
d) A zone of high pressure where simultaneous existence of high
tropical cold currents meet unemployment and skilled
labour
c) when additional labour do not
77.Consider the following contribute to the output
statements. d) when there are job vacancies
1. It rises in Tibet. in an industry despite
2. It enters India in Ladakh. producing full output
3. Shyok, Nubra and Hunza are
amongst its tributaries.
80. Which of the following
The above may refer to which river
roles/functions are performed by
system?
the High Court in India?
a) Indus 1. Determining the
b) Brahmaputra Constitutional validity of any
c) Luni
legislation or action of the
d) Gandak
executive in the country
78. In a water vessel, total 2. Giving judgments and
pressure exerted at the bottom of directives to protect public
a container depends on interest and human rights
1. Height at which the container throughout India
is placed above Mean Sea Which of the above is/are correct?
Level (MSL)
2. Height of water column a) 1 only
3. Width of water column b) 2 only
c) Both 1 and 2
Select the correct answer using the
d) None
codes below.
a) 1 and 2 only
http://www.insightsonindia.com INSIGHTS Page 17



81. The ‘Model Code of 84. Which of the following
Conduct (MCC)’ has been issued force(s) is/are both attractive and
by the repulsive?
a) Supreme Court of India 1. Gravitation Force
b) Election Commission of India 2. Electromagnetic force
c) President of India 3. Strong interaction force
d) Parliament
Select the correct answer using the
82. Which of the following codes below.
countries is/are NOT a part of G- a) 1 and 2 only
20 (Major economies)? b) 2 and 3 only
1. South Africa c) 1 and 3 only
2. Morocco d) 3 only
3. Mexico
4. Saudi Arabia
Select the correct answer using the 85. The Terai region is a
codes below. a) plain swampy landscape south
of Shiwaliks
a) 3 and 4 only b) highland east of Tibet Plateau
b) 4 only c) riverine island supporting
c) 2 only high biodiversity
d) 1 only d) hilly region in North-eastern
India
83. To become a law, all bills
passed by the Lok Sabha and 86. Extratropical storms are
Rajya Sabha must be approved different from Tropical storms in
by which way?
1. Speaker, Lok Sabha a) Moisture is carried in lower
2. Joint Parliamentary atmosphere in case of tropical
Committee storms, whereas it is carried in
3. Vice-President upper atmosphere in
4. President extratropical storm
b) tropical storms bring rain,
Select the correct answer using the while extratropical storms do
codes below. not
a) 1 and 3 only c) Extratropical storms are
b) 2 only caused due to easterly trade
c) 4 only winds, whereas tropical
d) 3 and 4 only storms are caused without any
trade winds
d) None of the above

http://www.insightsonindia.com INSIGHTS Page 18





90. ‘IMPRINT’ India initiative
87. China-Pakistan economic recently launched by the
corridor (CPEC) project seeks to President of India relates to
connect a) Boosting research in the
a) Chah bahar port in Pakistan to country
the Tibetan region occupied by b) Imparting skills to young
China graduates
b) Lahore in Pakistan to Beijing c) Promoting vernacular
in China literature in urban centres
c) Gwadar port in Pakistan to the d) Reviving traditional book
Xinjiang region in China presses in financial crisis
d) Bunker facilities in Islamabad,
Pakistan to shipping mouths 91.Thal, bhor and the pal ghats
near East China Sea passes relate to
a) Vindhya mountain ranges
88. Antyodaya Anna Yojana b) Eastern Ghats
(AAY) is a c) Western Ghats
a) Food for works programme d) Rajmahal Hills
b) subsidized food entitlements
scheme for Below Poverty Line 92. A constitution serves which
(BPL) citizens of following purposes?
c) food security financial support 1. It is the supreme law which
programme for determines the relationship
undernourished villages in among citizens living in a
India territory.
d) None of the above 2. It limits the powers of
government.
89. Which of the following can
Which of the above is/are correct?
be a low-cost technique(s) to
separate healthy seeds from a) 1 only
damaged seeds? b) 2 only
a) Treating the seeds with starch c) Both 1 and 2
b) Floating seeds on water d) None
c) Checking whether the seed has
sharp ends
d) All of the above

http://www.insightsonindia.com INSIGHTS Page 19





93. Consider the following Select the correct answer using the
about the Asian Infrastructure codes below.
Investment bank (AIIB).
a) 1 only
1. It admits only Asian member
b) 2 and 3 only
countries.
c) 3 only
2. It is led by China.
d) All of the above
3. All members have equal
voting rights in AIIB.
96. Consider the following with
Select the correct answer using the reference to ‘Over-the-top
codes below. services’ in India.
1. It is provided directly by the
a) 1 and 2 only Internet Service Providers
b) 2 only (ISPs).
c) 2 and 3 only 2. They are presently regulated
by the Ministry of Information
d) 1 and 3 only
& Broadcasting.
Which of the above is/are correct?
94. Mangroves do NOT occur a) 1 only
in which state(s) in India? b) 2 only
1. Andhra Pradesh c) Both 1 and 2
2. Goa d) None
3. Odisha
4. Meghalaya 97. Deen Dayal Upadhyay
Grameen Jyoti Yojana
Select the correct answer using the (DDUGJY) does NOT focus on
codes below. a) Providing electricity to all
a) 2 only villages in India
b) 1 and 3 only b) Separating feeder lines for
c) 2 and 4 only rural industrial and other uses
d) 4 only c) Reduction of transmission and
distribution losses
95. Which of the following d) Bridging revenue loss by
phenomenon can be described effective monitoring of
with the help of electricity usage
electromagnetism?
1. Friction
2. Rainbows
3. Lightning
http://www.insightsonindia.com INSIGHTS Page 20



98. The implementation of the 100. Consider the following
‘Smart Cities Mission’ at the City statements.
level will be done by a Special 1. It is located on the edge of the
Purpose Vehicle (SPV). Which of Eastern Himalaya biodiversity
the following statements is hotspot.
INCORRECT about SPVs? 2. It is a World Heritage Site
a) It will be a limited company 3. Vegetation consists of semi-
incorporated under the evergreen forests, tropical and
Companies Act. subtropical moist broadleaf
b) SPVs will not include any forests, Savannas and
private sector participation in shrublands.
ownership. 4. The park area is circumscribed
c) Government of India will by the Brahmaputra River.
provide funds to the SPVs to
The above refer to which National
implement the project.
park in India?
d) Both the State government
and Urban Local Body will a) Nokrek National Park
control the SPVs. b) Keibul Lamjao National Park
c) Namdapha National Park
d) Kaziranga National Park

99. Which of the following


statements is INCORRECT about
‘Home Guards’?
a) It is a voluntary force.
b) It assists the police forces in
maintaining internal security.
c) It serves as an auxiliary to the
Border Security Force (BSF).
d) It does not engage itself in
socio-economic and welfare
activities.

http://www.insightsonindia.com INSIGHTS Page 21

Das könnte Ihnen auch gefallen